You are on page 1of 99

UNP REVALIDA 2017

Table of Contents

1) Pediatrics
2) Histology
3) Bioethics
4) Pathology
5) PHAD
6) Physiology
7) Internal Medicine
8) Para and Micro
9) Legal Med
10) Anatomy
11) BICHEM (separate File BRS)
PEDIATRICS REVALIDA
2017

1. Most common genetic cause of moderate cognitive impairment with approximately 1 in


733 incidence in live births?
a. Trisomy 13
b. Trisomy 18
c. Trisomy 21
d. Trisomy 8
2. Most common neurobehavioral disorder affecting school-aged children
a. Dyslexia
b. Attention-deficit hyperactivity disorder(ADHD)
c. Autism
d. Asperger’s syndrome
3. Most common rheumatic disease inchildren
a. Juvenile Idiopathic Artheritis
b. SLE
c. Rheumatic Heart Disease
d. Scleroderma
4. Only available vaccine against Tuberculosis
a. DPT vaccine
b. Vericella vaccine
c. MMR
d. BCG
5. A major manisfestation of the rheumatic fever which is isolated frequently, neurologiv
behaverior disorder
a.
b. Cardiatis
c. Sydenham colera
d. Migratory poly arthraitis
6. Most common esophageal disorder in childern among allages
a. trachea esophageal fistula
b. esophagitis
c. Gastro esofageal reflux disease(GERD)
d. Esophageal Perforations
7. Complication of P. falciparum malaria manfested as hypotension, hypothermia, rapid
weak pulse, shallow breathing, pallor and vascular colapse
a. Topical spleenomegaly syndrome
b. Prostration
c. Algid malaria
d. Cerebral malaria
8. A chronic multisystem disorder caused by thopheryma wipphlet which encompasses
weight loss, diarrhea, abdominal pain, diarrhea, occulant bleeding from bowel mucosa,
hepatosplenomegaly, hepacitis andascitis
a. whipple disease
b. abetalipoprotenemia
c. topical sprue
d. autoimmune enteropathy
9. A condition where profound anemia becomes evident by 2-6 months of age associated
with growth retardation cranio facial and thumb abnormality
a. congenital dusentropoietic anemia
b. diamond blackfin anemia
c. megaloblastic anemia
d. pearson syndrome
10. A developmental disorder characterised by hypothyroidism, ptosis , short neck low set
ears short strature congenital heart disease and multiple skeletal and hematologic
abnormalities ?
a. Cartilage hai hypoplasia
b. Seckel syndrome
c. noonan syndrome
d. Dubowitz syndrome
11. most common cause of congenital adrenalhyperplasia
a. 11 beta hydroxylase deficiency
b. 21 hydroxylase deficiency
c. 17 hydroxylase deficiency
d. 3 beta hydroxysteroid dehydrogenasedeficiency
12. most common cause of congenitalhypothyroidism
a. defective synthesis of thyroxine
b. defect of iodine transport
c. thyroid dysgenesis
d. defect of thyroglobulin synthesis
13. major cause of bronchiolitis in children
a. RSV
b. influenza virus
c. parainfluenza virus
d. herpes virus 6
14. most common small vessel vasculitis inchildren?
a. SLE
b. rapidly progressive glomerulonephritis
c. henoch schonlein purpura
d. post streptococcalglomerulonephritis
15. Which is the deficient branch of amino acid chain in maple syrup urine disease (MSUD)?
a. Valine, leucine, isoleucine
b. Glycine
c. Phenylanaline
d. Tryptophan
16. New born birth weight- 3500 gm. What is an expected BW after 1 year?
a. 10.5kg
b. B. 8.5kg
c. 7.5kg
d. 11.5kg
17. Area of behaviour which includes pousturing of head, trunk, and extrimities, and well
coordinated movements of small movements (eg holding a pen or spoon or small
objects)
a. Language
b. Adaptive Behaviour
c. Motor
d. Personal and social
18.
19.
20.
21. Which is the deficient branch of amino acid chain in maple syrup urine disease (MSUD)?
a. Valine, leucine, isoleucine
b. Glycine
c. Phenylanaline
d. Tryptophan
22. New born birth weight- 3500 gm. What is an expected BW after 1 year?
a. 10.5kg
b. 8.5kg
c. 7.5kg
d. 11.5kg
23. IEM with X-linked mode of inheritance
a. CAH (congenital adeenalhyperplacia
b. G6PD
c. comgenital hypothyroidism
d. galactocemia
24. specific antidot for isoniazide toxicity
a. vit B1
b. vit B12
c. vit B6
d. vit. C
25. intracranial hemorrhage due to perinatal asphyxia, what is the grade of intraventricular
hemorrhage if germinal matrix isinvolved?
a. Grade 1
b. Grade 2
c. Grade 3
d. Grade 4
26. Antidote for paracetamol:
a. Vit B1
b. N-acetylcsteine 20%
c. Vit B6
d. S-methyl cysteine
27. The ingredients of wasuti (fire crackers) is highly toxic and can lead to painful death?
a. Yellow phosphorus
b. Red phosphorus
c. Green phosphorus
d. Blue phosphorus
28. Which of the followings are nontoxic EXCEPT?
a. Silica
b. Fabric softener
c. Hydrogen peroxide
d. Indelible markers
e. Nota
29. paralytic shellfish poisoning due to this toxin found in molluscum and fish consider
potent toxin in
a. teradotoxin
b. saxitoxin
c. sarombrotoxin
d. ciguatoxin
30. anti convulsant cause of neural tube defect, microcephaly, cardiac abnormality, various
bony and soft tissue :
a. valproic acid or valproate
31. whey:casein protein ratio of breastmilk
a. 40:60
b. 60:40
c. 70:30
d. 30:70
32. A baby is expected to have first tooth/teeth erupt at what age?
a. 6-8 month
b. 8-10mo
c. 10-12 mo
d. 1 yr
33. decrease cerebral blood flow due to meningitis result to ischemia, infarction, hypoxia.
Clinically these are symptoms manifested as
a. seizures
b. headache
c. body weakness
d. altered mental illness
e. a and b
f. c and d
34. PE findings where the skull when tap will produced a CRACKED POT sound
a. sun setting sign
b. brudzinski’s sign
c. macewen’s sign
d. kernig’s sign
35. IV duration for acute purulent meningitis
a. 7-10 days
b. 10-14days
c. 5-7 days
d. 3 weeks
36. Most common complication of Acute purulentmeningitis
a. cerebral abscess
b. cerebral edema
c. ventriculitis
d. subdural effusion
37. 12y old baby show muscle spasm and thus also show inability in chewing and drooling
PTC 1day And before first symptom appear the previous week the px. had a dental
carries and being operated for that thus px show
a. generalized tetnus
b. localized tetanus
c. neonatal tetanus
d. cepalic tetanus
38. The above px should be given the how much single dose of TIG via intramuscularly
a. 1500iu
b. 500iu
c. 50000iu
d. 10000iu
39. newborn at 5 minutes of life has the following : acrocynosis, CR > 100 per/min,
vigorously crying, inactive motion and sneezing. What is the apgar score of the patient
a. 10
b. 9
c. 8
d. 7
40. Adaptive Fine motor behaviour of a 10 month old year baby boy
a. Rakes at small objects
b. Throws toys
c. Waves bye-bye
d. Points to objects
41. A 15 month old female infant can do following, EXCEPT
a. Imitates household work such assweeping
b. Cripps upstairs
c. Walks backwards
d. Points to body parts
42. Risk factors which influence the vertical transmission of HIV infection includes the
following, EXCEPT
a. prematurity(<34 weeks AOG)
b. high maternal CD4 T-lymphocytescounts
c. prolong rupture of membranes (>4hrs)
d. use of recreational drugs duringpregnancy
43. Phase of HIV with highest turnover of HIV and CD4 T lymphocytes which leads to
gradual decline in immune system
a. acute retroviral syndrome
b. AIDS
c. clinical latency
d. NOTA
44. Most reliable indicator of disease progression in HIVinfection
a. CD8 T lymphocytes
b. CD4 T lymphocytes
c. gp120
d. gp41
45. CD4 T lymphocyte binding site in HIV genome..??
a. p24
b. gp41
c. p10
d. gp 120
e. all of the above
f. none of the above
46. Most common manner of HIV infection from mother to infant..?
a. utero placenta transmission
b. ingestion of infected breastmilk
c. passage thru infected vaginal canal
d. thru Caesarean section
47. A 4 YEAR OLD MALE CHILD WAS BROUGHT FOR CONSULT BECAUSE OF ACUTE
ATTERY DIARHEA OF 3 DAYS DURATION, THE FIRST STEP YOU WILL DO AT
THE ER IS
a. hook the patient at IV fluid
b. do the airway breathing circulation(ABC) assestment at once
c. prescribe zinc and give vitamin a capsule
d. classify the patient according ot the level of dehydration
48. what is the first line of treatment for patients 2-5 yrs old with bloody
diarhea(shigellosis)
a. ciprofloxacin
b. tetra cycline
c. metronidazone
d. tinidazole
49. pia 2 year old child have acute bloody diarrhea for 2 weeks with generlized abdomenal
pain and vomiting of previous ingested food and milk, intial consult was 12 days ago at
physician who give ciprofloxacin for 3 days, oral rehydration, salt solution, zinc
supplement and vitamin A capsule. The bloody stool resolved after treatment. 3 days
ago, however there was recurrence. So what is your clinical presentation of the diarrhea
in the patient?
a. acute watery diarrhea
b. acute bloody diarrhea
c. persistent diarrhea
d. diarrhea associated with malnutrition
50. miconutrient supplementation durinng the acute diarrheal episode include the
following except?
a. folic acid
b. iron
c. MgSO4
d. Zinc
e. all of the above
f. none of the above
51. true for periodic breathing
a. consist of apneaic pause 20-30sec pause followed by rapid breathing
b. more Common in preteen
c. Not common in term
d. AOTA
52. True for grunting, Except
a. Expiration through partially closed vocalcords
b. Low pitched inspirator sound
c. An attempt to increase airway pressure and lung volumn resulting to ventilation
perfusion V?F ratio
d. AOTA
53. RDS is suspected in newborn with respi. Distressif
a. cynotic, minimal or no response to 100% O2
b. Improved breathing and O2 response in first 24 hrs oflife
c. A&B
d. None
54. True for PPHN except n
a. ormal radiographic findings
b. PVR is increased afterdelivery
c. normal 2-D echo
d. Rarely improves after 24 hrs
55. true for hyalin mem. Dz
a. lung appear deep purple red liver likeconsistency
b. num.of alveoli duct,alveoli respiratory bronchioles are linedwd
acidophilia,homogenous or granularmem
c. the hyaline mem.are rare seen in infant
d. AOTA
56. true for mgnt of transient tachypnea of newborn
a. exogeous surfactant has no role inmanagement
b. endotracheal suctioning will shorten the clinicalcourse
c. a and b
d. NOTA
57. ventilation perfusion mismatch 8s more commonly in the FF
a. MAS
b. RDS
c. TPHN
d. AH
58. true except:
a. meconium stained amniotic fluid 10-15% commonly in preterm
b. 30% MAS mechanical ventilation
c. 3-5 % MAS expired
d. fetal distress, hypoxia before passage of meconium into amniotic fluid
59. true meconium passage
a. spped up surfactant
b. can cause lung atelectasis
c. routine resuction included endotrachialsuction
d. B & C only
60. true EXCEPT:
a. sign of intra utero fetal distress are fetal acidosis late deceleration of poor beat
variabing
b. amniotic infusion does not reduce risk of meconium aspiration syndrome
c. routine intratracheal patern nasopharynx suctioning does not reduce risk of
MAS
d. NOTA
61. A non edematous child with weight for age 55% has...
a. Undernutrition
b. Marasmus
c. kwashiorkor
d. marasmus with kwashiorkor
62. A edematous child with weight for age 55% has...
a. undernutrition
b. marasmus
c. kwashiorkor
d. marasmus with kwashiorkor
63. Non edematous child with weight for age80%
a. Undernutrition
b. Kwashiorkor
c. marasmic
d. kwashiorkor
e. marasmus
64. Edematous child weight for age 75%
a. Options are same as above question
65. A non-edematous child with weight for age of 65% has:
a. undernutrition
b. Kwashiorkor
c. Marasmic Kwashiorkor
d. Marasmus
66. true regarding the F100 diet;
a. 419 mOsmol/L
b. 0.9 g of protein per 100ml solution
c. contains copper of 0.15mg per 100mlsolution
d. NOTA
67. True for F75 solution
a. Contains 420 kj from 1L of solution
b. Contains 1.3 gm of calcium per 1L
c. Contains 1.9 mmol sodium per 1L
d. NOTA
68. Which of the following is true regarding grownpattern?
a. Child fall in normal percentile development pattern is normal growth
b. Growth is a stasis process
c. Child would lose 10% of weight during first 2wks of life and regain it during 6th
wk of development. And child would countuie to gain 20gm-30gm per day until
3rd month of life.
69. Signifies cumulative effects ofundernutrition;
a. weight for height
b. height for age
c. weight for age
d. basal metabolic rate
70. For FTT include
a. an impatient evaluation on all cases of FTT
b. the child can be observed for caloric intake, daily weight change, intraction with
the parent/caregivers and behaviour duringfeeding
c. A and B
d. None of the above
71. 12years old baby was brought for consult due to complain of undernutrition. Patients
weight was 28kg and height was 45inch. What is the ideal weight
a. 31kg
b. 35kg
c. 40kg
d. 45kg
72. What is the ideal height in inches for thispatient
a. 45 inches
b. 48 inches
c. 60 inches
d. 65 inches
73. snoring,mouth breathing,enlarged tonsils in children with FTT means
a. organic cause of FTT
b. psychosocial cause of FTT
c. GERD
d. milk production
74. assessment of food intake?
a. Qualitative
b. Quantitative
c. 3 days diet record
d. all of the above
75. common causes of irregular growth curvesexcept:
a. improperway of measuring theanthropometrics
b. error in plotong the mesurements
c. wrong growth chart
d. none of the above
76. organic causes of FTT, EXCEPT:
a. zinc defeciency
b. lead toxicity
c. family dysfunction
d. AOTA
77. Proper way of measuring child'sheight...
a. face,shoulder,and feet should touch/face the vertical space
b. slight upward traction can be applied under the child's chin to avoid "turtling" of
nech when the fixed Headpiece of infantometer comes in contact with the top of
child's head.
c. AOTA
78. True about growth:-
a. growth is process
b. in evaluating growth of child,growth velocity or weight gain over a period of
time is more important than attained lenght,height orweight
c. single measurement of length,height or weight is of limited use
d. AOTA
79. A child with weight and height falling below the 5th percentile and dysmorphic facial
fetures could most likely have
a. Endocrine disorder
b. Organic FTT
c. Chrosomal disorder
d. All of the above
80. True for the measurement of anthropometric EXCEPT
a. Head circumferance is determined using a flexible tape measure run from the
supraorbital ridge to the occiput
b. In a 4yr old child, length is most accurately measured by two examinies (one
osition the child), with the child supine
c. A stadiometer is used to measure the height ofchildren
d. NOTA
81. TRUE FOR ANALYSIS OF GROWTHPATTERN
a. GROWTH IS A STATIC PROCESS
b. Weight is due to genetic makeup ofgene
c. SIZE AT 6 MONTHS CORRELATES WITH MEAN PERCENTAGEHEIGHT
REFLECTING INFLUENCE OF GENE
d. AT THE AGE OF 2-3 YEARS OLD CHILDS PATTERN GOES DIRECTLY UPWARD
OR DOWNWARD
82. FTT IS .....
a. MUST TREATED IMMEDIATELY
b. PERCENTILE UNDER 5TH LINE
c. INPATIENT MANAGEMENT WARRANTED IN ALLCASES
d. NONE OF THE ABOVE
83. What expected height for 3 years old child who born at 35 weeks AOG?
a. 85
b. 90
c. 95
d. None of the above
84. Which of the following is true?
a. Laboratory test require in all cases of FTT
b. Inpatient management is require in all cases of FTT
c. FTT is a diagnosis that requires immediatelytreatment
d. None of the above
85. Mother Brought 8 day old infant, she felt that her daughter is not getting enough
breastmilk. PE was normal with present weight of 2.7kgs. What will you advise the px?
a. Findings are normal
b. Further evaluation should be done
c. Advise supplement milk formula
d. Advice appetite stimulants
86. What is the expected weight of the one month old Child with birth weight of 3 kg
a. 3.1
b. 3.2
c. 3.4
d. 3.6
87. True for adolcents, Except
a. dengue is one of the top casing admissions among adolscents in philippines
b. hetrosexuals face greater barriers to sexual health services
c. pregnancy complications is common in teenagepregnancies
d. sexual behaviour does not equate sexualorientation
88. true for risk taking behaviour among adolscents, except
a. applies to any behaviour that can compromise adolscent development
b. risktaking is necessary to delevop selfconfidence
c. risk taking among the adolscents can bebenfiical
d. NOTA
89. Accurate tanner staging is required for all pediatrics patients, during adolscent
interview and physicalexamination
a. Assure absolute confidentiality
b. Always assume that adolscent engage in only hetrosexual relationships
c. CRAFT questions is a brief screening test of adolscent for eating disorder
d. None of the Above
90. True for eating disorders in adolscents, EXCEPT
a. BMI bellow the 2nd percentile is an indication for hospitalization in patients with
aneroxia nervosa
b. Disturbance in the neuro transmittor serotonin might be a risk factor for
developing anorexia nervosa
c. Criteria for the diagnosis of the bulumia is the amenorrhea for 3 consecutive
cycles in previously menstruating adolscent
d. NOTA
91. Pedgree show/s;
a. female are more affected
b. mother with the disease are unlikly to have offsprings with the disease
c. an affected father will have no affectedoffspring
d. AOTA
92. In this type of genetic inheritance
a. the affected mother may be asymptomatic
b. exibit atosomal dominant form of inheritance
c. y-linked form of inheritance
d. AOTA
93. This pedgree shows the:
a. classic form of mendelian inheritance
b. Autosomal dominant form of inheritane
c. Y-linked form of inheritance
d. NOTA
94. Common manifestation/s with this type of heridity is/are;
a. developmental delay
b. seizures
c. decreased strength and tone
d. AOTA
95. In the 2nd generation of 5 siblings
a. 3 sibling got married
b. all siblings has disease
c. male has 3 affected child
d. none of the above

96. pedigree shows


a. skipped generation
b. autosomal recessive mode oftransmission
c. females are the carrier
d. none of the above
97. This mode of inheritence shown in thepedgree
a. most often associated with femaleinfertility
b. it is rare to see familialtransmission
c. A,B
d. None of the above
98. Horizontal transmission is seen/ observedwith;
a. multiple affected members of kindered in the same generation
b. no affected family members in othergeneration
c. A,B
d. NOTA
99. True For Mendelian pattern of inheritance:
a. Foundation of single gene inheritance
b. Explain heredofamilial diseases
c. Pattern of inheritance in diabetes
d. NOTA
100. Neural tube defects can be preventedby:
a. Intake of iron supplement duringpregnancy
b. Intake of calcium supplement duringpregnancy
c. Intake of folic acid for 3 months afterconception
d. NOTA
Histology Revalida
2017

1. Patient with chronic burning sensation in chest which is worsening with lying
down after meal at night and relieved with antacid. Most common structural
dysfunction in this patient?
a. Pancreas
b. Pylorus
c. Upper esophageal sphincter
d. Lower esophageal sphincter
2. what is the signal for release of CKK from small intestine to blood?
a. Increase gastric motility
b. Bile acid in ileum
c. Fatty acid in duodenum
d. Acid in duodenum
3. which of the following is neurotransmitter, hormones, paracrine substance has
greatest impact on acid secretion by parietal cell of gastric mucosa ??
a. histamine
b. acetylcholine
c. gastrin
d. somatostatin
4. 22 year old university students, Ms. Smith has complaint with central
abdominal pain.. Pain increased in severity and localized right lower abdomen
with nausea and vomiting and she was taken to ER. Her body temperature
was 38 degree C at the time of admission. What is the diagnosis?
a. Acute cholecystitis
b. Acute appendicitis
c. Biliary colic
d. Peptic ulcer
5. A patient present with the recurrent fatty diarrhea, weakness, anemia,
malabsorption for 6momths. Which of the following gastro-intestinal structure
is responsible for absorption and digestion:-
a. Oral cavity
b. Esophagus or gullets
c. Stomach
d. Small intestine
6. Which of the following statement concerning the histological structure in the
fundus/body region of the stomach is correct:-
a. long pits and very short gland found
b. Parential cells are mainly located at the base of gastric glands and chief
cells are mainly located at the neck region of the gastric gland
c. chief cell produces pepsin
d. Gastric gland do not contains APUD
7. saliva passes through straight duct ,which are the following changes occur
a. active secretion of Na+
b. active secretion of water
c. active secretion of Cl-
d. absorption of Na+
8. which are the following characteristic of gastric mucosa
a. primary protection layer
b. muscularis is additional layer
c. submucosa is infiltrate with inflammation
d. somatic innervation
9. true about enteroendothelial cells (EEC)
a. peneath cells are EEC
b. EEC in stomach secreted by mucosal cell
c. no EEC cells in dudenum
d. G cells are EEC &amp; secreted by
10.not secreted by submandibular gland
a. mucous acinus
b. serous acinus
c. intercalated duct
d. centroacinar cell
11. Permanent mucosal fold in the wall of small intestines?
a. valve of Houston
b. valve of kerckring
c. plicae transversalis
d. haustrae
12. centroacinar cells are found in which of the following glands?
a. parotid
b. submaxillary
c. sublingual
d. pancreas
13. which of the following histological feature is important in preventing
mechanical erosion of the esophageal surface during swallowing?
a. non keratinized squamous epithelium and mucus gland
b. skeletal muscle in upper third
c. smooth muscle in lower third
d. mixure of skeletal and smooth muscle in middle third
14. this histological structure in large intestine is responsible for efficient
peristalsis?
a. haustra
b. plicae semilunares
c. intesinal gland
d. taenia coli
15. Myelination process of the CNS accompanied by
a. Astrocytes
b. Oligodendrocytes
c. Schwann cell
d. Neural crest cell
16. episodic demyelination disease is characteristic of
a. Parkinson&#39;s disease
b. Meningitis
c. Multiple sclerosis
d. Huntington chorea
17. acetylcholine is only neurotransmitter of nervous system
a. central nervous station
b. motor cortex
c. presynaptic sensory cortex
d. motor nervous system
18.characteristics of synaptic vesicles
a. Release of nor-epinephrine
b. synthesis of nor-epinephrine
c. incorporated in presynaptic membrane
d. enter to synaptic cleft
19. Myelination process of the CNS accompanied by
a. Astrocytes
b. Oligodendrocytes
c. Schwann cell
d. Neural crest cell
20. episodic demyelination disease is characteristic of
a. Parkinson&#39;s disease
b. Meningitis
c. Multiple sclerosis
d. Huntington chorea
21. Myelination of peripheral nerve is accompanied by:
a. Astrocytes
b. Oligodendrocyte
c. Neural crest cell
d. Schwann cell
22.Purkunjie cell is found in
a. Retina
b. Cerebrum
c. Cerebellum
d. Cranial nerve
23. Most common neuronal type
a. Unipolar
b. Bipolar
c. Multi polar
d. Pseudomultipolar
24.Intimate covering of the brain
a. Sharpey’s fibers
b. Arachnoid matter
c. Pia matter
d. Dura matter
25. which of the following is component of peripheral nervous system?
a. Cerebellum
b. Spinal cord
c. Dorsal root column
d. Cranial nerves
26. what is the space between liver sinusoid and hepatocyte, called
a. Howships canal
b. Peri-sinusoidal space
c. Disse
d. Lacunae
27. Which neuron is multipolar?
a. motor neuron
b. ganglion
c. Olfactory neuron
d. sensory neuron
28. Which is correct sequence of salivary secretion?
a. Intercalated duct>striated duct>acinus>excretory duct
b. Acinus > intercalated duct> striated duct >excretory duct
c. Excretory duct >intercalated duct>striated duct >acinus
d. Intercalated duct > striated duct >acinus> excretory duct
29.what is cell body of neuron called
a. ganglion
b. astrocyte
c. soma ?
d. nissl ( ans)
30. where is ependyma is located
a. lining vertebrae
b. adherent to brain
c. covering spinal cord
d. covering brain
31. what type of papillae on the tongue is the most abundant ?
a. filiform papillae
b. circumflex papillae
c. fungiform papillae
d. foliate papillae
32. what type of papillae on the tongue is not well developed in men?
a. filiform papillae
b. circumflex papillae
c. fungiform papillae
d. foliate papillae
33. What type of papillae are not found in taste bud
a. Filiform
b. Circumvallate
c. Folliate
d. Fungiform
34. Lymphatic capillary present in villous of small intestine?
a. Lacteal
b. Plica transversis
c. Thoracic duct
d. Crypts of leiburkehm
35.exocrine portion of pancreas
a. islets of Langerhans
b. alpha cells
c. beta cells
d. acini
36. Diverticula of gall bladder is called
a. Rokitansky aschoff sinus
b. B. duct of luschka
c. C. duct of wirsung
d. D. ampulla of vater
37. Functional unit of liver
a. lobes
b. portal triad
c. central vein
d. hepatocytes
38.Somatostatin is released by
a. Langerhans cell
b. alpha cells
c. beta cells
d. delta cells
39.
40.
41. Layer not found in gallbladder ?
A submucosa absent in gallbladder
B mucosa
C lamina propria
D adventitia
42. Lining of thyroid follicle ?
A simple cuboidal epithelium
B columnar
C stratifier sqamus
D simple sqamus
43. What cells produce pepsinogen ?
a. oxyntic cells
b. parietal cells
c. chief cells
d. neck mucosal cells
44. Of what layer is Meissner’s plexus is a part of ?
a. mucosa
b. submucosa
c. muscularis externa
d. serosa
45. Type of the tissue line in upper esophagus..?
a. Simple cuboidal
b. Simple squamous
c. Simple columnar
d. Stratified squamous
46. modification of muscularis externa seen on large intestine
a. strait border
b. plicae semilunares
c. Taenia coli
d. crypts of linder
47. Which of the following is the major type in LI epithelium?
a. Columnar absorptive cell.
b. Parietal cell
c. Paneth cell
d. Goblet cell
48. Goblet cells are most numerous in:
a. Esophagus
b. Stomach
c. SI
d. LI
49. Alveoli lined by?
a. simple columnar epithelium
b. simple squamous epithelium
c. simple cuboidal epithelium
d. transitional epithelium
50.Hallmark for cardiac myocyte?
a. striated muscle
b. gap junction
c. multi nucleated
d. involuntary
UNIVERSITY OF NORTHERN PHILIPPINES 4) justification or homologus
COLLEGE OF MEDICINE insemination except:
WRITTEN REVALIDA – 2016/2017 A) oligospermia
BIOETHICS B) defect in quality of the sperm of
husband
C) husband is impotent
D) spinal injury of the husband that
1) stem cell research is put into
hinders normal intercourse
question by Christian ethics because
E) all of the above
of following reasons
F) none of the above
A) it involves manipulation of human
embryo or fetus
5) this is process where a human
B) it is laboratories fertilization
conception is obtain to invitro
C) it is a form of IVF which involves
fertilization to generative cells taken
conception outside the worm by
from the at least one donor often then
artificial means
the two spouses in marriage
D) it is a crime against the dignity as
A) cloning
human being
B) hetarologus in-Vitro fertilization and
E) all of the above
embryo transfer
F) none of the above
C) hetarologus artificial insemination
D) homologus artificial insemination
2) potential of stem cell research
expect:
6) justification for artificial insemination
A) stem cells can replace damage
procedure
cells of the brain and spinal cord
A) every couple has the right to
B) it can replace damage cell or death
overcome childlessness by whatever
cells of the kidney, lungs, and liver
means of re-production
C) stem cells can be induced to
B) artificial insemination produces a
become pancreatic cells then
child who is not a fruit of conjugal act
transplanted to the pancreas to cure
C) it transfers procreation into a
diabetes
biological laboratory
D) stem cells can replicate very fast
D) unitive meaning of conjugal love is
E) all of the above
preserved
F) none of the above
7) principals involved in organ
3) this procedure consist of depositing
donation and transplantation
a man’s semen in the vagina, cervical
A) moral principal of totality
canal or uterus through the use of
B) it is against human dignity
instrumentation to bring about
C) a human decision of sharing an
conception an attainable by sexual
organ to those who are needing it
intercourse
D) it is against organ commercialism
A) stem cell transplant
under the pretext of volunteer organ
B) invitro fertilization
donation
C) artificial insemination
E) all of the above
D) cloning
F) none of the above
8) Christian ethics only endorses this states that an individual right to cut off,
type of contraception mutilate or remove any defective or
A) natural method of responsible non functioning part of his body
parenthood A) principle of double effect
B) permanent surgical contraception B) principle of stewardship
C) intrauterine device C) principle of totality
D) use of artificial contraception to D) inviolability of life
prevent HIV
E) all of the above 12) the deliberate expulsion of a living
F) none of the above fetus from the mother’s womb before
the age of viability violate this Christian
9) this is deliberately including the principle of ethics
expulsion of a living fetus in order to A) principle of double effect
save the life of mother from the B) principle of stewardship
dangers brought about by pregnancy C) principle of totality
A) natural abortion D) principle of inviolability of life
B) eugenic abortion
C) abortion of fetal indication 13) as mandated by Christian duties,
D) therapeutic abortion we have the right to preserve the
environment and trusted to us by the
10) the principle of double effect creator the practice of reduced, re-use
applies to situations in which a good and recycle uphold this principle of
effect and an evil effect (which is ethics
permitted to occur but not directly A) principle of double effect
intended) will result from a good B) principle of stewardship
cause, however conditions must be C) principle of totality
fulfilled to considered the act morally D) principle of inviolability of life
legitimate. such conditions include the
following expect 14) the principle of double effect
A) the action directly intended must be applies to the following situation
good in itself except
B) the good effect must follow from the A) Expulsion of a live fetus before the
action at least immediately as the evil age of viability because parents are
effect not legally married
C) the foreseen effect must be B) removal of cancerous uterus in
intended or approved from the very pregnant patient
beginning C) removal of fallopian tube or part of it
D) there must be proportionate and due to ectopic pregnancy
sufficient reason for allowing the evil D) subjecting a mentally incapacitated
effect to occur while performing the women to permanent surgical
action sterilization after delivery because it
was found out that the pregnancy was
11) Christian ethics mandates that we due to incest by her step father
have a natural obligation to preserve
our lives and preserve the integrity of 15) humility is one of the most
our body. There for, this principle important virtues of a medical doctor,
this virtue is exemplified in the B) voluntariness
following situations C) understanding
A) doctors bragging about once D) comprehension
expertise to impress the students E) All of the above
B) encouraging students to have F) none of the above
additional opportunities for learning
C) doctors continuously updating once 20) in non emergency situations when
knowledge through continuing a person is incompetent to make a
professional development lectures decision (two sick or two immature) the
D) telling the patient about the truth decision and consent is made by a
and nature of his/her illness substitute decision – maker who can
be following except
16) valid permission for specific A) common law wife
procedures in health care research B) legal wife
and educations C) grand-mother (legal guardian)
A) respect for person D) legal eldest son-daughter
B) free and informed consent E) All of the above
C) principle of non maleficence F) none of the above
D) principle of justice
21) “premium non-nocere” is a
17) the principle of solidarity is applied negative action exemplified in the
in following situation except: following situations except
A) doctor acting kindly and A) providing incompetent care
compassionately toward his patient B) respecting dignity
B) member of health care team C) breaching privacy
advocating just health policies for the D) causing hopelessness
marginalized E) All of the above
C) use of psycho, pharmaceutical and F) none of the above
psychotherapy
D) volunteering to be an organ donor 22) Beneficence is a bioethical
principle demonstrated in the following
18) the principle of inviolability of life is situations expect
violated in following situations expect A) becoming a role model to others
A) upholding death penalty for heinous B) guiding a younger colleague
crimes C) proving a free medical service
B) physician assisted euthanasia D) donating a charity institution
C) natural abortion E) All of the above
D) suicidal bombing F) none of the above

19) this is the ability of the person 23) Christian principles of ethics that is
giving consent to receive the violated in organ donation
information, remember, understand A) stewardship
and access the data then use it to B) totality
make and enlightened decision based C) non maleficence
on a rational reason D) beneficence
A) competence E) A & C
F) B & D 29) Christian principle of ethics applied
in ghost surgery permitted by the
24) characteristics of situations director of hospital
surrounding “ a good death” A) principle of double effect
A) a basic need met without feeling B) principle of legitimate cooperation
dependent or burden to others C) principle of totality
B) well cared for by loved ones D) principle of stewardship
C) must be amidst familiar surrounding
D) pain managed well 30) which moral maximum is
E) All of the above demonstrated in the “robinhood
F) none of the above concept”
A) no one is obligated to betray himself
25) the physician, spouse, a close B) an object cries out for its owner
friend of nearest of kin besides that the C) the end does not justify the means
life of terminally – ill patient should be D) the salvation of the soul is the
terminated highest law
A) active and non voluntary
B) active and voluntary
C) passive and non voluntary True & False :
D) passive and voluntary write “ X ” if statement is correct
write “ O ” if statement is incorrect
26) Christian principle of ethics applied
in correction of anatomical in born 1) to develop the appropriate the
defect secondary sex characteristics, both
A) principle of double effect male and female are given sex
B) principle of stewardship hormone of their new gender
C) principle of totality
D) principle of inviolability of life 2) true biological sex is not changes
despite sex changed procedure
27) Christian principle of ethics
initiating mangrove planting in a flood 3) behavioural signs tell us that sexual
prone coastal town where fishing is the thoughts and fantasies serve as a
primary source of livelihood means by which people deal with
A) principle of double effect minor sexual tensions that might
B) principle of stewardship otherwise build up and released in
C) principle of subsidiarity more explicit manor
D) principle of totality
4) development in changing cultural
28) Christian principle of ethics applied valuation as to what constitute
in the inclusion of sexual education in maleness or femaleness indicates
school curriculum preferences for homosexuality
A) principle of personalized sexuality
B) principle of stewardship 5) the female homosexual is
C) principle of subsidiarity stereotyped as a feminine appearing
D) principle of totality women marked by certain preferences
in dress, job and appearance
6) sexuality is not inherent in all aspect 20) there is natural inclination of an
of human life individual to self preservation at all
costs.
7) organ donation and transplantation
are good an should be encouraged

8) a donor – cadaver can not violate


the principle of non-maleficence

9) direct sterilization for non medical


reason is not morally justifiable

10) intercourse for fun or pleasure


seems impossible to morally justify the
act

11) marriage is still marriage even


when there are no children

12) the church and society have the


right to bar the adoption of children by
homosexual couples.

13) heterosexual act are not free from


ontic evil

14) after legal separation, both parties


can not re-marry

15) after divorce, both parties can re-


marry

16) prolife advocates maintain that the


unborn fetus has equal right as an
adult, thus abortion is never
permissible

17) the homosexual acts involve a


significant degree of ontic evil because
of their lack of openness to procreation

18) cause of suicide can be due to


financial, personal or social causes

19) mercy killing violate the principle of


stewardship an inviolability of life
UNP-COLLEGE OF MEDICINE
WRITTEN REVALIDA 7.) In hypothyroidism, all of the following are true
except:
MARCH 28, 2017
a) Plasma T3 may be in normal range
CLINICAL & SYSTEMIC b) Plasma t4 may be in normal range
PATHOLOGY c) Thyroidal radioactive-iodine uptake is
always low
d) Plasma TSH may be low, normal or
1.) All but which of the following will effectively prevent increased.
the loss of glucose in whole blood 8.) In normal subjects, administration of
a) Sodium oxalate thyrotropin releasing hormone stimulates the:
b) Sodium fluoride a) Prolactin
c) Prompt centrifugation b) Growth hormone
d) Prompt refrigeration c) Thyroid stimulating hormone
2.) One of the following conditions lead to d) Luteinizing hormone
hypoalbuminemia except: 9.) The differential cell count in neonates differs
a) Liver disease from that in adults to preponderance of:
b) Infection a) Lymphocytes
c) Severe burns b) Monocytes
d) Dehydration c) Neutrophils
3.) An elevated alkaline phosphatase along with the d) Neuroectodermal cells
normal transaminase results is most characteristic of 10.) Which of the following would not be expected
uncomplicated: to cause Xanthochromia?
a) Hepatitis a) Epidural hemorrhage
b) Cirrhosis b) Traumatic tap
c) Obstructive jaundice c) Jaundice
d) Erythroblastosis d) Markedly elevated protein
4.) Enzymes acts by: 11.) Following hemorrhage in the CSF, when would
a) Shifting the equilibrium position of a you expect typical xanthochromia to appear:
reaction a) 2-12 hours
b) Causing the thermodynamically b) 1-3 days
incompatible to occur c) 12-24 hours
c) Increasing the rate of reaction in one d) After 1 week
direction 12.)Which anticoagulant should be used for the
d) Decreasing the amount of free energy collection of blood gases:
of activation necessary a) Heparin
5.) Serum alkaline phosphates is most commonly b) Oxalate
elevated in disease of: c) EDTA
a) Pancreas and liver d) Citrate
b) Liver and kidney 13.)What is the most important buffer in the urine:
c) Intestine and bone a) Bicarbonate
d) Liver and bone b) Phosphate
6.) The enzyme most sensitive to the liver cell damage, c) Ammonium
such as hepatitis, is: d) Protein
a) ALT 14.)Which test is the best indicator of bicarbonate
b) Alkaline phosphates level:
c) LD a) pCO2
d) GGT b) total co2
c) buffer base
d) Co2 combining power
d.) Phospholipids
23.)Man is the definitive host of all but one of which
15.)Which of the following phospholipid is the most
of the following:
sensitive and specific indicator of pulmonary
a.) D latum
maturation
b.) E granulosus
a) Phosphatidyl glycerol
c.) T saginata
b) Phosphatidyl inositol
d.) A lumbricoides
c) Phosphatidyl serine
24.)The infective stage of plasmodium sp. From the
d) Phosphatidyl ethanoiamine
mosquito is:
16.)What is the maximum permissible time period
a) Merozoite
between whole blood collection and platelet
b) Sporozoite
concentrate:
c) Gametocyte
a) 4 hours
d) Trophozoite
b) 6 hours
25.)This type of cast represent the final stage of
c) 8 hours
disintegrated cellular casts:
d) 12 hours
a.) Finely granular cast
17.) Which class of immunoglobulins has the most
b.) Hyaline cast
efficient agglutinating capacity
c.) Fatty cast
a) IgA
d.) Waxy cast
b) IgG
26.)The smallest tapeworm of man is :
c) IgM
a) Hymenolepsis nana
d) IgE
b) Echinococcus granulosus
18.)Which class of immunoglobulins most
c) Taenia saginata
effectively fixes complement:
d) Diphylobothrium latum
a) IgG
27.)Ascaris lumbricoides adult worms have a
b) IgM
predilection for the:
c) IgA
a.) Liver
d) IgE
b.) Small intestine
19.)Staphylococcus aureus is different from
c.) Large intestine
S.epidermidis:
d.) Perianal folds
a) Coagulase
28.)The presence of red blood cells casts in the
b) Hemolysis
urine sediment may indicate all of the following
c) Novobiocin resistance
except:
d) Phosphatase
a.) Lipoid nephrosis
20.)The most common cause of acute cystitis is
b.) Crescentic glomerulonephritis
a) Klebsiella Pneumonia
c.) Acute interstitial nephritis
b) Escherichia coli
d.) Acute proliferative lupus nephritis
c) Mycoplasma hominis
29.) Which of the following would favor cast
d) Chlamydia trachomatis
formations in the urine:
21.)The antigenic substances that react with the
a.) High potassium concentration
antibody causing transfusion reaction is located
b.) Urine ph
on the surface of the:
Kindly c.) Osmolality
a) WBC
check ans d.) Urine stasis
21 b) RBS
30.)This crystal seen in acidic urine
c) PLATELET
a.) Calcium phosphate
d) AOTA
b.) Calcium carbonate
22.)Steatosis describe abnormal accumulation of
c.) Calcium oxalate
the substance with in parenchymal cells:
d.) Ammonium biurates
a.) Cholesterol
b.) Triglyceride
c.) Lipoprotein
39.) The normal reticulocyte count of the new born is :
31.) One of the following elevated in patient with a.) 2-4%
leukemia b. ) 2-6%
a.) Urea nitrogen c.) 2-8%
b.) Uric acid d.) 2-10%
c.) Cholesterol 40.) The color of normal urine is primarily to the
d.) Calcium oxalate presence of:
a.) Prophyrins
32.) Turbidity of CSF may be caused by an increase
b.) Urobillinogen
a.) RBC count c.) Urochrome
b.) WBC count d.) Billrubin
c.) Both
d.) Neither 41.) The most common cause of non specific vaginitis:
33.) An adult man previously treated for tuberculosis
a) Trichomonas
may given blood
b) Gardnarella
a.) 6 months after recovery
c) Monilia
b.) 12 month after recovery
d) Herpes simplex
c.) 2 years after recovery
d.) Never 42.) An epileptic patient can never donate blood:
34.) The shortest duration for blood donation
a) True
a.) 2 months
b) False
b.) 3 months
c.) 4 months 43.) Anti- acne medication containing retinoic acid
d.) 5 months dictate that users donate blood:
35.) Patients taking metronidazole develop a:
a) 1 month after cessation of treatment
a.) Yellow brown urine
b.) Red brown urine b) 2 months after cessation of treatment
c.) Orange brown urine c) 3 months after cessation of treatment
d.) Brick brown urine d) 6 months after cessation of treatment
36.) The L/S ratio reaches a maturation level normally 44.) An 30 year old female on oral contraceptive can
at: donate blood
a.) 20 wks AOG
b.) 30 wks AOG a) Anytime
c.) 25 wks AOG b) 1 month after cessation of intake
d.) 35 wks AOG c) Right after menstruation
37.) Amount of protien in the urine in persistent d) At menopause
proteinuria : 45.) Which of the following assess the intrinsic factor
a.) 1-2g/day
b.) 2-3g/day a) INR
c. ) 2-4g/day b) Protime
d .) 4-5 g/day c) Partial thromboplastine time
38.) Disseminated intravascular coagulation (DIC) d) Bleeding time
freguently accompains liver diseases causing an 46.) Which of the following etiologic agent is screened
increased consumption of:
during blood donation?
a.) Clotting factors
b.) Platelet a) Picorna virus
c.) Ammonia b) Delta virus
d.) AOTA c) Calci virus
d) Hepadna virus
47.) Which of the following is accepted for blood 54.) The urinary bladder musculature in a male with
donation? chronic urethral obstruction will undergo which type of
adaptation?
a) 96.8 Lbs donor
b) Donated on November 1 a) atrophy
c) Temp of 38 c b) hypertrophy
d) Cancer survivor c) hyperplasia
d) metaplasia
48.) Coffin lid urine crystal:

a) Uric acid 55.) This pigment result from from the oxidation of
b) Calcium oxalate tyrosine to dihydroxyphenaylalanine
c) Triple phosphate
a) Lipofuchsin
d) Leucine
b) hemosiderin
49.) CSF leukemic cells determination should use: c) billrubin
d)melanin
a) 1st tube
b) 2nd tube
c) 3rd tube 56.) Which of the following is/are chemotactic factors
d) 4th tube below.

50.) The doctor wants to monitor for recurrence of coon a) bacterial products
carcinoma. Which of the marker is useful: b) cytokines
c) complement
a) CA 125 d) all of the above
b) CEA
c) CA 15-3
d) CA 19-9 57.) Which of the following is most important for the
process of phagocytosis?
SYSTEMIC PATHOLOGY:
a) Receptor sites
51.) Duplication of DNA occurs in which phase of the
b) Complement
cell cycle?
c) Antibodies
a.) G0 d) phagocytic vacuole
b.) G1
c.) G2 58.) Granulation tissue is characterized by:
d.) S
a) proliferation of capillaries
52.) Which of the following lesions is/are associated b) loose edematous stroma
with chronic tuberculosis? c) presence of leukocytic cell infiltrates
a.) T-lymphocytes accumulation d) all of the above
b.)macrophage transformation 59.) Swelling of soft tissue around a subcutaneous
c.) central ischemia of lesion abscess is due to:
d.) all of the above
a) Increased hydrostatic pressure
b) diminished osmotic pressure
53.) Leukocytes are classified as: c) lymphatic obstruction
a) labile cells d) increased vascular permeability
b) stable cells
c) temporary cells
d) permanent cells
60.) The following are usually found in acute 67.) Brain of 70 yrs old lady has progressive memory
inflammation? loss and with significant non occlusive atherosclerosis
are found with ds lesions:
a.) neutrophil infiltration
b.) fibrous tissue proliferation a.) Atrophy
c.) liquefactive necrosis b.) Liquefactive necrosis
d.) vascualar congestion c.) Chronic inflammation
d.) AOTA
61.) Suppuration is due to.
68.) Which of the following lesions found in
a.) vascular insuffiency
atherosclerosis of aorta?
b.) poorly digestable inorganic irritants
c.) pyogenic organism a.) Necrosis
d.) protein denaturation b.) Calcification
c.) Lipid accumulation
62.) A granuloma is formed by:
d.) AOTA
A.) neoplastic lymphocytes
69.) Mitral stenosis is most commonly due to:
B.) granulation tissue
C.) caseous necrosis
D.) epitheloid histiocytes a.) congenital valvular defect
b.) rheumatoid heart dse
63.) Viruses elicits which kind of immune response c.) degenerative senile changes
A.) CELL MEDIATED d.) myxomatous valvular degeneration
B.) HUMORAL 70.) Which lesion is associated with prolapsing leaflets:
C.) BOTH a.) congenital valvular defect
D.) NEITHER b.) rheumatoid heart dse
64.) The hepatic changes in passive congestion are seen c.) degenerative senile changes
in which of the following structure d.) myxomatous valvular degeneration

a.) A. hepatic artery 71.) Asymmetric thickening of the uterine musculature


b.) hepatic vein that present with punture hemorrhages and gland
c.) billary duct secretion
d.) central vein and adjacent sinusoids a.) Endrometrial polyp
65.) A mass of disorganized but mature cells indigenous b.) Adenomyosis
to a particular site is called: c.) Intramural leiomyoma
d.) Intramural leiomyosacroma

a.) teratoma 72.) Best prognostic factor for breast Ca.


b.) hamartoma a.) histologic type
c.) chorangioma b.) Hormonal receptor status
d.) choriostoma c.) Tumor size
66.) Which of the following is a mismatch pair? d.) Status of the lymph node

a.) HPV – squamous cell ca of cervix 73.)The most common cancer of the breast is derived
b.) ionizing radiation – malignant melanoma from the
c.) Aspergillus flavus – hepatocellularca a.) lactiferous duct
d.) nitrosamines – adenocancer of stomach b.) nipple
c.) fibroadipose stroma
d.) terminal duct lobular unit Not confirm
74.) Most common sarcoma occurring in childhood and 81.) Which of the following primary malignant breast
adolescence? tumors ofter the best prognosis assuming clinical stage
of the same?
a.) Rhabdomyosarcoma
b.) Leiomyosarcoma a) Medullary carcinoma
c.) Liposarcoma b) Invasive ductal carcinoma
d.) Fibrosarcoma c) Infiltrating lobular carcinoma
d) Angiosarcoma
75.)Which of the following is bone tumors is composed
of stromal cells and osteoclast like cell 82.) The histologic basis of a high grade phyllodes tumor
is based on
a.) Osteosarcoma
b.) Chondrosarcoma a) degree of anaplasia involving the
c.) giant cell tumor epithelial component
d.) metastatic tumor b) degree of anaplasia involving the
stromal component
76.) Most common testicular tumor in children upto 3
c) degree of anaplasia involving the both
yrs old
epithelial and stromal component
a.) EST (endodermal sinus tumor) d) presence of metastasis
b.) Choriosarcoma
83.) Eosinophilic Mallory bodies are seen in
c.) Teratoma
d.) Seminoma a) alcoholic hepatitis
b) autoimmune hepatitis
77.) Testicular analogue of dysgerminoma of the ovary?
c) chronic hepatitis
a.) Seminoma d) acute hepatitis
b.) Choriocarcinoma
84.) What is expected to be complicated by pulmonary
c.) sertoli-leydig tumor
hypertension?
d.) embryonal carcinoma
a) Tetralogy of fallot
78.) Histologic section of testicular tumor shows
b) Severe pulmonary stenosis Not sure
squamous epithelium, fat, bone and island of glial
c) Ventricular septal defect
elements. The diagnosis is ?
d) Cor pulmonale
a.) seminoma
85.) The earliest morphologic evidence of pulmonary
b.) embryonal carcinoma
hypertension is
c.) teratoma
d.) EST (endodermal sinus tumor) a) Medial hypertrophy
b) Intimal hyperplasia
79.) The most reliable criterion for the diagnosis of
c) Plexiform lesion
leomyosarcoma of uterus is
d) Intimal fibrosis
a.) Cellularity
86.) Alcohol induced liver injury is frequently associated
b.) Hemorrhage
with which of the following
c.) nuclear atypia
d.) mitotic count a) Councilman bodies
b) Eosinophilic Mallory bodies
80.) In Bethesda system, an adequate smear means
c) Piecemeal necrosis
a. The presence of endocervical cell for evaluation d) Steatosis
b. the absence of endocervical cell
c. the presence of superficial and intermediates
but no endocervical component
d. scanty cellular smears
87.) The findings of patchy collections of lipid laden 93.) Most common tumor of appendix
macrophages in the lamina propria of the gallbladder is
A. Squamous cell carcinoma B. Carcinoid tumor
seen in
C. Neuroblastoma D.Signet ring carcinoma
a) Acute calculous cholecystitis
b) Chronic calculous cholecystitis 94.) A tuberculosis lesion in the intestine would be seen
c) Cholesterolosis grossly as
d) Mucocoele
a. Ulcer perpendicular to the long axis of the
88.) The usual histologic involving a primary malignancy intestine
of the extrahepatic bile duct would be b. Nodules along the long axis of the intestine
c. Presence of flask shaped ulcer
a) Adenocarcinoma
d. Fungating mass at the ileocecal area
b) Lymphoma
c) Sarcoma 95.) The histologic feature is seen in crohn’s disease but
d) Squamous cell carcinoma not in ulcerative colitis
89.) An 8 year old girl presents with a unilateral ovarian a. Presence of mixed inflammatory infiltrate
mass with a clinical presentation of precocious puberty. b. Presence of clusters with inflammatory infiltrate
The expected predominant cell type that will be seen in c. Presence of ulcers
her vaginal smear would be: d. Presence of dysplastic epithelium
a) Intermediates 96.) H.pylori is associated with following lesion EXCEPT:
b) Parabasals
c) Superficials A. Chronic gastritis B. Gastric carcinoma
d) All of the above C. Peptic ulcer D. Achalasia

90.) Which among the following condition offers the 97.) TRUE of Meckel’s diverticulum:
best prognosis for the primary breast cancer? a. Its due to failure of involution of the vitelline
a) Positive metastasis involving 7 of 20 axillary duct
lymph nodes b. Seen mostly in the mesenteric border of the
b) Negative expression of her2/ness and p53 intestine
c) Invasive ductal carcinoma c. Measures around 2cm in length
d) Mucinous carcinoma with negative axillary d. A false diverticulum
lymph node involvement 98.) The most common malignancy affecting the bone
91.) The feature that most clearly differentiates primary a. Osteosarcoma b. Multiple myeloma
from secondary healing c. Chondrosarcoma d. Metastatic carcinoma
a) Granulation tissue 99.) Which one of the following is associated with
b) Presence of macrophage pickwickian syndrome?
c) Contraction
d) Number of inflammatory infiltrate A. Kwashiorkor B. Marasmus
Not sure C.Obesity D. Bulema
92.) These lesions can be result of trypanosoma cruzi
infection 100.) The following conditions can cause an obstructive
type of atelectasis, EXCEPT:
a) Esophagitis
b) Achalasia a. Excessive bronchial secretions
c) Traction diverticulum b. Foreign body aspirations
d) Barrett disease c. Effusions from cardiac failure
d. Bronchial neoplasm
WRITTEN REVALIDA – 2016/2017
Family & Community Medicine 5. Which of the following is included in the
"extended family?
1. What is the branch of Medicine that deals
with provision of comprehensive health care A. Sister-in-Haw
to people regardless of age or sex? B. illegitimate child
C. Step-parents
A. Public Health D. Foster children
B. Community Medicine
C. Preventive Medicine
D. Family Medicine
6. What would consist tertiary level of
prevention for the family?
2. Component of the family health care plan
A. Hospice care
which involves health maintenance for each
B. Health education
family member:
C. Counseling
D. Breast examination
A. Preventive
B. Diagnostic
C. Therapeutic
D. Rehabilitative 7. What is the start of the family life cycle?

A. Newly married couple


3. The following ls/are important role/s of a B. Family with young children
family physician: C. Unattached young adult
D. Family with adolescents
A. Personal physician to each member of the
family
B. Manages the collective health problems of 8. What does a dashed line(----) that connects
the family a male and female couple in the family
c. Recognizes the effects of illness upon the bgenogram denote?
other family members
A. married couple
D. All of the above
B. consanguineous marriage
C. legally separated couple
D. live-in couple
4. As parents, Mr. and Mrs. A make sure that
they spend quality time with their children.
They play and do things together with them.
9. Who is the first of the community to the
What function of the family do they perform?
health chain, as defined by the PHC system?

A. Biologic A. village health workers


B. Educational B. intermediate level health workers
C. Psychologic C. hospital personnel
D. socio-cultural D. barangay captain
10. The initial step in community planning is:
15. What is the best screening procedure for
A formation of objectives tuberculosis in an "under six” program?
B. identification of roles and their
A. Chest x-ray
relationships
B. Sputum AFB
C. educational analysis
C. Tuberculin test
D. determination of resources
D. Sputum culture

11. The core list of drugs approved and


authorized by the Department of Health is 16. In the routine immunization of children,
the: the following is NOT included:

A. BCG
A. Generic List
B. Typhoid
B. Complementary Drug List
C. Essential Drug list C. Measles
D. Diphtheria
D. Prohibited Drug list

12. The most effective control measure for 17. The leading cause of maternal death in the
Tuberculosis in the Philippines is Philippines is:

A. Abortion
A. Mass BCG vaccinatlon
B. Toxemia of pregnancy
B. "Clean Air" campaign
C. Infection
C. Massive nutritional correction and support
D. Hemorrhage
D. Promotion of chemoprophylaxis for
contacts

18, Immunization is contraindicated In:


13. What is NOT part of the Family Planning A. malnourished children
program? B, children with diarrhea
A. Promotion of knowledge on the use of c, children with prolonged febrile illness
contraceptive devices D. breastfed babies
B. Helping childless couples to bear children
C. Encouraging legal abortion
D. Providing family planning services 19. What is the most effective method of
contraception?
14. What is the toxic effect of isoniazid?
A. PUD
A, diarrhea B. Rhythm method
B, hepatitis C. Pills
C, chorioretinitis D. Barrier or condoms
D, extrapyramidal symptoms
20, what Is NOT a mosquito-borne disease? 25. A 20 Year-old female In respiratory
distress was brought to the emergency room
A. Malaria
for intentional ingestion of poisonous
B. Dengue
substance of the following, which is the most
C. Tuberculosis
appropriate thing to do first.
D. Filariasis
A. Get detailed history
B. Maintain vital signs
21, According to the Expanded Program of C. Give antidote
Immunization of the Philippines, BCG vaccine D. Induce vomiting
is given at:

A. 3 months
26. Contamination of food by cockroaches has
B. 9 months
example of
C. 6 month
D, at birth A. vehicle borne transmission
B. air-borne transmission
C. mechanical transmission
22. Ability to stimulate the host to produce D. biological transmission
agglutinin, opsonins, antitoxins, etc,

A. Pathogenicity
27. Gas responsible for explosion in garbage
B. virulence
dumps:
C. Antigenicity
D. Infectlvity A. carbonic acid gas
B. methane
C. carbon monoxide
23. A fully immunized child has finished: D.nitrogen

A. BCG, 2 DPT, 20PV


B. BCG, 3 DPT, 3 OPV For questions #28-29, refer to the facts below:
C. BCG, 3DPT, 30PV, MEASLES Data from clinical studies are reported
D. 3 DPT, 30PV, MEASLES frequently in the form of 2 by 2 tables. A 2by 2
table for a hypothetical test shown below:

24. There Disease are internationally Test Result Disease Disease


Present Absent
quarantine disease and thus are labeled as
Positive 50 20
Diseases under Surveillance by WHO:
Negative 30 50
A. plague, cholera, smallpox
B. plague, typhoid fever, cholera 28. What is the test sensitivity?
A. 63%
C. leprosy, smallpox, plague
B. 71%
D. malaria, laprosy, smallpox
C. 29%
D. 38%
29. What is the positive predictive value? 34. objective of tertiary prevention of
A. 71% communicable disease include
B. 47%
C. 38% A. removal of the agent from the environment
D. 33% B. help the patient function maximally within
the restriction imposed by the disease
C. Screening
30. Increase in life expectancy is mainly due D. Immunization
to:

A Decrease in mortality in the younger age 35. The primary objective of health education
groups is:
B. Improvement in health information
dissemination A. impart knowledge
C. Better diagnostic facilities B. promote health in general
D. Discovery of effective drugs c. improve health practices
D. transfer technology to lay personnel

31. What is considered the most dangerous


possible disease spread? 36. The greatest contributor to population
growth in ASIA today is:
A, clinical stage patient
B. non-immunized A. Increased mortality rate
C. clinical carrier B. Increased birth rate
D. terminally ill patient C. Decreased birth rate
D. Increased mortality rate

32. What is focal occult blood a screening test


for? 37. life expectancy is lower among the

A. colorectal cancer A. Married


B. cervical cancer B. Single
C. hemorrhoids C. Young professionals
D. Inflammatory bowel disease D. Separated/divorced

33. What is the process of isolating patients 38. The rate that measures the probability of
with Flu called? a person dying in a year regardless of age, sex,
race, etc, is:
A. Specific protection
B. Early diagnosis and treatment A. case fatality rate
C. Disability limitation B. neonatal mortality rate
D. Hearth promotion C. Crude death rate
D. Specific death rate
39. What does the Expanded Program of 43. What is a very early sign of dehydration in
immunization (EPI)of the government aim to a child with diarrhea?
decrease the prevalence of?
A. thirst
A. tuberculosis, diphtheria, pertussis, tetanus, B. increased pulse and breathing rates
polio, measles, Hepatitis A C. Sunken eyes
B. tuberculosis, diphtheria, pertussis, tetanus, D. somnolence
polio, measles, Hepatitis B
C. tuberculosis, diphtheria, pertussis, tetanus,
polio, German measles, Hepatitis B 44. Phenomenon where in there is occurrence
D. tuberculosis, diphtheria, Hepatitis B of illnesses of similar nature at a rate above
tetanus, polio, measles, Hepatitis A the expected number:

A. hyperendemic
B. epidemic
40. Which of the following is/are
C. Endemic
characteristic/s of passive immunity?
D. Pandemic
A. it is temporary
B. it usually lasts for 6 months In the newborn
C.it protects an individual for life 45. Where are births in the Philippines
D. A and B only registered.

A. Where the infant's parents live


B. where the Infant was born
41. Majority of pregnant Filipino women
C. of the parents' choice
suffer from:
D. listed as the family's permanent residence
A. anemia
B. scurvy
C. Vitamin deficiency 46. An investigator wanted to determine if
D. Lack of iodine there is a significant difference in the mean
systolic blood pressure of 10 women before
and after contraceptive use for 3 months.
42. What is removed in the chlorination of What s the appropriate statistical test for
water? significance for this study?

A bad taste A. Analysis of variance


B. Turbidity B. Chi-square analysis
C. Odor C. Student t-test
D. Bacteria D. Paired t-test
47. When distribution is symmetrical, what is
the measure of central tendency to use?

A. Mode
B. Median
C. Mean
D. All

48. Graphical presentation of data most


useful in locating cases during an epidemic:

A. Spot map
B. Pictograph
C. Scatter diagram
D. Histogram

49. which of the following sampling methods


is appropriate when a population is not widely
spread geographically?

A. Random
B. Multi stage
C, Stratified
D. Systematic

50. What is the best screening procedure for


tuberculosis in an under six" program?

A. Chest x-ray
B. Sputum examination for AFB
C. Tuberculin test
D. B and C only
D. Thyroid hormone synthesis become excessive as a
result of increase iodine exposure

University of Northern Philippines C) Biosynthesis Effect caused by incapacity


Written Revalida – 2016/2017 thyroid to secreted adequate amount of
hormone leading to increasing TSH and larger
Physiology
goiter

1) Hormone secreted by thyrotrope cells and


5) Which among the following thyroid
considered as most useful marker of thyroid
hormone has half life of the 7hrs and is
hormone action
directly produced from the thyroid hormone?
A. TSH
A.Thyroxine
B. TRH
B.Triidothyronine
C. Thyroxin
C.Reverse Triidothyronine
D. Triiodothyronine
D.Thyroid stimulating hormone

2)which Sequel of thyroid hormone synthesis


?? 6) During pregnancy, total thyroid hormone
A.iodide trapping - >organification->coupling- increase, reflection increase
>release A.thyroxine binding globulin
B.Organification->Coupling->Iodide trapping- B.Thyroid stimulating hormone
>Release C.folicular stimulating hormone
C.Iodide trapping->Coupling->Organification- D. Leutenising hormone
>release 7 Ans
7) :- B.thyroxine binding globulin
D.coupling->Organification->Iodide trapping-
8)effect of thyroid hormone :
>release
A. decrease basal metabolic rate
B. decrease protein, lipid, and glycogen
3) True of thyroid hormone trapping.
synthesis
A. Uptake of iodide from the circulating is
C. Permissive effect on growth, development
through pendrin located the basal surface of
and fertility
thyrocytes.
D. Decrease cell responsiveness to
B. Uptake of iodide from the circulating is
catecholamines
through sodium iodide symporter located the
basal surface of thyrocytes.
C. mutation of sodium iodide symporter gene 9) which is True about adrenal cortex ?
caused by syndrome with hyperthyroidism, A.the left adrenal vein drains directly into the
goiter and deafness inferior vena cava and the right adrenal vein
D. Effuse of iodide to colloid is through drains from right renal vein to IVC
sodium iodide symporter located at apical B.The cortex is divided into 3 layers(inner to
border of thyroid follicle outer)Glomerulosa,fasciculate,reticularis
C.the zona glomerulosa has aldosterone
synthase which converts corticosterone to
4) What is Jod Basedow ?
aldosterone.
A. Displacement of bound thyroid hormone
D. zona reticularis aldosterone synthase which
thus increasing free hormone pool
D B) Excess of iodide exposure transientlly by
converts corticosterone to aldosterone.

inhibits iodide magnification decreasing


thyroid hormone synthesis.
10) mineralocorticoids stimulants caused by D.They are located anterior to the right and
RAA/ renin system inhibited by except ?? left thyroid glands
A. ACTH
B. macula densa feedback on sodium 16)Parathyroid hormone effects:
resorption A.Indirect effect on the intestine via 1,25 Vit.D
increasing interstitial calcium absorbance
C. renal perfusion pressure
D. serum sodium B. Direct effect on bone decreasing calcium
resorption
C.Increases plasma phosphorus
D.decreases renal calcium reabsorbance
11) hormone release by corticotrope cells
and regulate adrenal cortisol synthesis?? 17) True of parathyroid hormone:
A. ACTH A.single chain protein containing 27 amino
B. CRH acid
C. Androgen B.the biologic activity of the parathyroid
D. deoxycorticosterone hormone residues in the carboxyl terminal
C.Hail life of PTH is 10 hours
D.its biological activity residues in amino
12) Precursor of all adrenal terminal
steroidogenesis???
A. progesterone 18)True of vit D metabolism :
D B. testosterone A.In the kidney ,24 hydroxylase activates 25
C. cholesterol hydroxy calciferol to 24,25
D. corticotropine Corticosterone dehydroxycolecalciferol
B.Vit.D3 is converted to 25 hydroxy vit.D(25-
OH Vit.D) in the kidney
13) What is the effect of aldosterone? C. 25 hydroxy vit.D(25-OH Vit.D) or calcitriol is
A. decrease arteria perfusion pressure the major circulating and storage form of the
B. potassium excretion vitamin D
C. sodium excretion D. the half life of 25hydroxy vitamin D is 2-3
D. potasdium retension days

14) inactivation of cortisol to cortisone by this


enzyme prevents mineralocorticoid receptor 19) True for the vitamin D
activation by axcess cortisol. A. Decrease plasma calcium
A. 11 B hydroxysteroid dehydrogenase type 2 B. Decrease plasma phosphate
B. 11 B hydroxysteroid dehydrogenase type 1 C. Decrease intestinal calcium absorption
D. Increase osteoid mineralization
C. aldosterone synthase
D. 3B hydroxysteroid dehydrogenase

15)True of parathyroid glands:


20) Not true for calcitonin
A.the two superior parathyroid glands can
A.clinically it can be used as tumor marker for
variably located in the neck and mediastinum
medullary thyroid cancer
B.The chief cells secrets the parathyroid
B.the parafolicular C cells of the thyroid gland
hormone
produce calcitonin
C. the parafollicular cells produce the
C.It place a very significant role in the calcium
parathyroid hormone
phosphorus metabolism
D.in the bone reticularis inhibits osteoclast
26) TRUE for cell membrane
mediated bone resorption
A. contains few proteins
B. Contain variable protiens and lipid
21)The type of blood flow when blood flows
depending on location in cell
as a steady rate through a long smooth blood C. stable arrangement according to cell life
vessel with each layer of blood remaining the D. freely permeable to electrolyte not to
same distacnce from the vessel wall protein
A.laminar flow
B.turbulent flow
27) Which is true for second messenger?
C.streaminase flow
A. Are substances that interact with primary
D. BOTH A and B
messenger at outside of cell
22)major factors affect the pulse pressure: B. Are hormones produce by cell to stimulate
A.stroke volume output of the heart other hormones
B.The compliance C. Intracellular interaction with
C. total distensiblity neurotransmitters
D. all of the above D. Are not form by brain?
Formed
E. a and b both
F. b and c
28) TheGolgi complex
A. Break down of lipid and protein
23) Damping of pulses caused by B. Energy production All options starts with
Is a organelle that participates
A. resistance of the vessel C. Protein processing in
B. Compliance of the vessel D. Accumulation of protein for nucleus
C. afterload tariffing
D. A & B
29) the 32 year old female received an
E. B & C
injection with local anesthetic for a tooth
F. AOTA
extractions, within 2 hours she noted Ans
palpitations, diaphoresis, and dizziness which B
24) Factors that increase venous return which of the following ionich channel correctly
increase right atrial pressure match with a component of the action
A. Increase blood volume potential:
B. Vascular tone that increase peripheral
A.
venous resistance
Ans 30 D .all are correct
C. dilatation of arteriole that increase
30) Options are like inversely related to ion conc
peripheral vascular resistance Direct related to the iron fat solubility
D. A & B
E. B & C 31. which one of the substance transported
F. AOTA by active transport except
A. NA
25) SPECIFIC BLOOD RESERVOIER” in the B. K
body. C. glucose
A. spleen
D. H
B. liver
C. Large abdominal vein 32. the following statement is true
D. venousplexsus under skin
A. binding to K carrier is associated with
E. AOTA
32 ans option c
ATP applied to the inside of the membrane
has no effect on the transport process

dephosphorylation 38) Greatest amount of cellular men


B. na/k pump inhibited by oubain excitability
C. ATP binding inside of the membrane has A. increase Extracellular K
B. increase extracellular Na
no effect on diffuse process
C. decrease extracellular Ca
D. 3 NA extrudes exchange with 2 k taken D. decrease extracellular Cl
into cell
39)which of the following statement is true:
Located
33) Na-k ATPase is a protein present in ? A.The the cell propogating nature of the
action potential due to circular current flow
A. nucleus
along the excitable fiber
B. RER B.during after repolzarization the threshold is
C. cytoso increase.
D. cell membrane C. the action potential does not obey the all
or none law.
34) What is generated k-reflux following Na+ D.during action potential polarity of the
reflux membrane is maintained.
A. resting membrane potential
40)when a cell is stimulated above the
B. hyperpolarization
threshold it becomes for a time unexcitable
C. depolarization what is this period called ?
D. repolarization A. relative refractory period
B. cell death
C.Repolarized period
D.Absolute refractory period
35)Movementof water from low to high
concentrationThrough semi permeable 41)in the generation of action potential there
membrane: is a moment of the transmembrane potential
A.osmosis towers the equilibrium potential for:
B.solvent drag A.sodium
B.k
C.diffusion
C.cloride
D. Facilitated diffusion
D.Calcium

42)During the rising phase of action potential


the nerve cell membrane is:
36) Membrane potential said to be positive if A. polarized
A.( -)and (+) charges equal on both sides of B.Very excitable
membrane C. Hyperpolarized
B.(-) charges equal on both sides D.Depolarized
C.(+)charges equal on both sides of The magnitude of the
membrane 43. An upstroke in an action potential is
dependent on ?
D. Different number of (-)& (+)charges on
A. increased sodium inside A.extracellular Na concentration
both sides of membrane A
B. increased potassium outside
C. diamter of axon
37) Threshold stimulate refers to : D. aota/nota
A.stimulant of just sufficient strange, which
44.An severe increase in calcium
affects a visible response.
concentrations above normal can cause what
B.least perceptible increase in the intensity of
of the following ?
the stimulus
A. tetany
C.
46. Ans B immylinated fiber needed
47. Ans B contain fatty acid
48.Ans B couple to metabolic enzyme expenditure

B. increased excitability of nervous system A. Substances that interact with 1st messenger
c.decreased QT interval outside the cell 53. Ans D
d. increased influx of NA B. Substances that interact with the 1st
messenger in the cell membrane
45)the velocity of the propogation along and
C. Hormones excreted by cells in response to
axion will increase if there is a decrease in other hormones
the: D.Mediate the intercellular responses to many diff.
A.Membranic capacitance> Hormones and neuro transmitter
54) True about endocytosis
B. Axon excitability A. Pinocytosis/Phagocytosis but not clarithin-
C.Membrane resistance mediated or claveolae-dependentuptkae of
D.Axon diameter extracellular material
46)
B. Merging of intracellular vesicles to plasma
49. anion concentrstion higer outwadr than membrane resulting in transfer of intracellular
inward. Permeability is dependenton which? substance to extracellular mileu
A. electical charge on anion
B. directly on fatty permeability C. Invagination of intracellular membrane for
C. intracellular anion concentration uptake of extracellular material
D. all of the above
D. Trafficking through golgi apparatus

50. water goes into cell by?


A. exocytosis 55) channolopathay involve mutation
B. osmosis of.calciun channel by extreme elevation of
C. active transfer core body temp.
D. endocytosis A.hyperthermia
B.barrter synd.
C.long Q T interval
51. Na+k+ pump is important because D.cystic fibrosis

A. Reduce intracellular k+ concentration


B. Facilitate na+ entry
56) inhibitory to na. K ATpase
C. Prevents cells from swelling and bursting
A. thyroid hrmn.
D. Produces positive charge in cell
B aldosteron
52.) The electronic na+k+ pump plays a critical C. insuline
role in cellular physiology by: D. dopamine
A. Using ATP to extrude 3 Na out and 2 K in
cells
B. Using ATP to extrude 3 Na in and 2 K out 57) scuba diver is at 132 FT and rise after 10
C. Use energy to extrude 3na out and 2k in to min and feels ill effect ; if he is underwater at
make ATP 540min and 90 ft he is advise to period of 720
D. Use ATP to extrude 3 Na in and 2k out to min of decompression before rise , otherwise
generate energy risk of bends is which
53) True about secondary messengers

C option for 52
using energy in moving Na into the cell
and K inside the cell make ATP
A. N2 is principle cause 64) One climber reaches to 4500m and
B. longer N2 exposure increase bend barometric pressure is 447mmHg so what will
C. N2 diffuse rapidly be PO2 of moisture?
D. N2 diffuse into tissue and to blood A. 47 B. 84 C. 63 D. 75

65)which of the following is not future of a


58) decompression sickness, symptoms by acclimatization to high altitude?
bubble of A. Hyperventilation
A. O2 B. CO2 C. H D. N2 B.Polycythemia
C.Increase in the number of capillaries
D.right ward shift of the o2 dissociation curve
59) the main cause of symptom experience
66)which of the following is incorrect?
by diver suffering from bends is ?
A.It can increase the oxygen consumption
A. oxygen toxixity
more than 10 fold compared with rest>
B. increase size if N2 bubble
B.The change in the ventilation on exercise
C. metabolic alkalosis
can be explained by fall in the arterial PH
D. severe respiratory acidosis
C.Ventilation increases more than cardiac
output
60) which of the FF is symptom of DCS ? D.At high level of exercise blood lactate
A. paralysis and respiratory failure concentration may rapidly increase
B. Severe joint pain
C. paresthenia and itching 67. Mixture of helium-oxide is used than
D. AOTA mixture of nitrus-oxide during deep diving to
reduce all of the following except:
61. what is most paO2 moist inspired gas of A. Work of breathing
climate on the summit of mt. everest [assume Ans B. Risk of oxygen toxicity
that bronchial pressure is 247mmHg] C C. Increase gas necrosis Risk of inert gas necrosis
A 32
B 52 D. Risk of decompression sickness
C 42
D 62
68. Absence of gravity during spaceflight
62. in zone 2 of lung all of the following
alters following except:
statement are true Except :-
A blood flow determined by arterial pressure A. Work of breathing
minus venous pressure B. Distribution of Pulmonary
B arterial pressure exceed alveolar pressure C. Thoracic blood flow
C arterial pressure exceed venous pressure D. Deposition of inhaled aerosol particles
D alveolar pressure exceed venous pressure
69 the following is increase by the percentage
63) The reason for hypoxia which is not at maximal exercise compare to the rest?
responded by oxygen treatment A. Heart rate
A. V/Q mismatch B. cardiac output
B. Decreased oxygen C. Alveolar ventilation
C. shunt D.Tidal volume
D. Anemia
C. potassium channel open facilitating
70) 35 y/o female present with weakness of potassium efflux
extra ocular muscle and weakness of D. the inferior of the channel in hypopotential
extremites which is releave by rest and
increase by physical activity she is treated 75)the termination of muscle contraction is
brought about the onset of:
with acetylcholine and get immediate relief
diagnosis A. potassium channel opening thus promoting
entry into the cell
A. Lambert teton syndrome
B. sodium ion entering the cell
B.Myasthenia Gravis
C.depolarization of the cell membrane
C.Multiple sclerosis
D. Closing of the sodium channel
D.Muscular dystrophy Calcium
76)an engorged or distended internal jugular
vein is an reflective of an increase pressure in
71) Nociceptors
which chamber of the heart ?
A.are activated by strong pressure, severe
cold,severe heat and chemicals A. Left atrium
B.Right atrium
B.are absent in visceral organs
C. Left ventricle
C.are specialized structures located on the
D. right ventricle
skin and joints
D.are innervated by group 2 afferents
E.are involved in acute but not chronic pain
77) Blood draining from head & neck entering
72) Generator potential : cardiac circulation via...
A. always lead to an action potential A. Superior vena cava
B. Increase in amplitude as a more intense B. Right atrium
stimulus is applied C. Inferior vena cava
C. Is an all or non phenomenon D.Pulmonic valve
D. Is unchanged when a given stimulus is
applied repetedly over time 78) a patient who has cardiogenic shock due
E.all of the above to severe dehydration will benefit with which
treatment strategy ?
73)characteristics of phase 2 A. Use of inotropics to raise the blood
pressure
A. bulk sodium influx B.Administration of isotonic IV fluids
B. fast opening of sodium channels C.Antibiotics
C. calcium influx enters calcium influx D.Digitalis to increase heart pumping ability
D. muscle relaxation
79)you are asked to manage a patient who
has hypertensive cardiovascular disease based
on the ECG tracing you show inversion of T
74)events occur during phase-0 of action
waves on the leads V4 to V6. Which part of
potential
the heart is involved ?
A. sodium channels open facilitating sodium
A. lateral wall
influx
B.Septum
B. calcium channel open allowing calcium
C.Inferior wall
channel enter
D.Posterior wall
80)your patient has acute myocardial wall
involving almost entire left ventricle you 85) Two-thirds of the thoracic cavity
expect to see the blocked of perfusion on the expansion is caused by
coronary angiogram along the territory of : A. Contraction of diaphragm
A.Right coronary artery B. Contraction of the scalenes
B.Left coronary artery C. Contraction of pectoralis minor
C.Left circumflex artery D. Release of surfactant
D.Left anterior descending artery
86) Muscle mostly used in heavy inspiration?
81) FUNCTION OF THE RESPIRATORY SYSTEM A. Diaphragm
A. ALLOW O2 AND CO2 CIRCULATION B. Scalene
B. PROTECTION FROM FOREIGN BODY C. Internal intercostal
C. AIR VIBRATING THE VOCAL CORDS CREATES D. External intercostals
SOUND
D. CAN ALTER THE PH BY CHNAGING O2
CONCENTRATION ALTERNATION 87) amount of air inspired or expired in a
E. A,B,C normal inhalation or exhalation is called
F. ALL OF THE ABOVE and has a volume of about ml.
A.Tidal volume,4600
82) FUNCTION OF THE NASAL CAVITY B.Vital capacity, 4600
A. FILTER THE AIR C.Residual volume,1200
B. HUMLSIFIED THE AIR D.Tidal volume,500
C. WARM THE AIR 88) largest air passage that engage in gas
D.EXTERNAL SITE FOR O2 AND CO2 exchange with the blood are..
E. A,B,C A.respiratory bronchioles
F. ALL OF THE ABOVE
B.terminal bronchioles
C.Primary bronchi
83) Point at which in respiratory tree where C
D.alveolar ducts
shape cartilage end up and from smooth E.Alveoli
muscle airway under the cartilage?
A. Primary bronchi
B. Terminal bronchioles 89) during heavy exercise there is increase
C. Carina flow to lung 4-7 times is due to
A.increase opening of capillaries 3 times
D. Respiratory bronchioles
B.distented of capillaries more than 2 times
Secondary bronchi
lead to increase blood flow
C.increase pulmonary arterial pressure
84) The protective barrier of the lungs, the D. All of the above
surfactant is produce by this pneumocyte?
A. Acinar 90.Bronchial circulation nourishes
B. Type I pneumocyte. A.tranchea
C.Type II pneumocyte. B.bronchi
D. Macrophages C.terminal bronchi
D.pleura
E. All of the above 97) increase of AP diameter during inspiration
is achieved by:
A.scalene muscle of the neck fix or immobilize
91) Functional anatomic unit t of lung ? the first pair of ribs
A.Bronchopulmonary segment B.contraction of the external intercostals
B.Region of the lung supplied by tertiary or muscle
segmental bronchus C.Contraction of the rectus abdominis
C.Respiratory bronchioles D. a b and c
D.All of above E. a and b
E. A and B
98)the pressure of given quantity of the gas is
inversely proportional to its volume (assuming
92 ) function of pleura ? the constant temperature)
A. lubrication A. Dalton’s law
B. in the mediastinum act as barrier between B. Henry’s law
cardiac cavity and plural cavity C. Boil’s law
C. pressure gradient D. Charles’ law
D. all
E. a or b only
Left 99) During quiet expiration
93) the characteristics of lungs include: A. Decrease thoracic volume by decreasing
A. Divided into upper middle and lower lobe pressure and relax diaphragm lung and alveoli
B.Interlobular fissure includes oblique and recoil
horizontal fissures B. Alveolar pressure higher than bariatric
C. Interlobular fissure include oblique fissure pressure, the air go out
only
C. Alveoli air decrease subsequently decrease
D. A and B pressure
94) Ans E. Both A and B D. Only A, B, C

100) which of the following cause dilation of


95) which one is dead anatomic space from the bronchioles
the following A. Bronchoconstriction
A. cconductive zone B. Parasympathetic
B. bronchi C. Histamine
D. Release of Epinephrine and norepinephrine From adrenal
C. respiratory bronchioles lacking alveoli medula
D. all are correct E. All of the above
E. a and b

96) which has diameter less than 1 mm


A. tertiary bronchi
B. terminal bronchi
C. respiratory bronchioles
D. all of the above
E. b and c
Internal Medicine

1. Sclera have a particular affinity for billirubin due to their high elastin content.
The presence of sclera icterus indicates a serum billirubin level of at least:
A. 1.3mg/dl
B. 3mg/dl
C. 1.3g/dl
D. 3g/dl
2. The most common and most characteristic symptom of liver disease is :
A. Fatigue
B. Jaundice
C. abdominal pain
D. abdominal enlargement

3. True regarding Hepatitis A infection EXCEPT:


A. Classified under hepatovirus
B. Will lead into chronicity
C. A positive igM Anti HAV signifies current infection
D. Supportive treatment is the main treatment

4. The mainstay of treatment in Non alcoholic fatty liver disease is:


A. Weight loss dietary pills
B. Exercise
C. Low calorie diet
D. Anti cholesterol drugs

5. Acute cholangitis is a dreaded complication of choledocholithiasis. It manifest


usually as
charcot’s triad w/c comprises of the following EXCEPT:
A. Right upper pain
B. Jaundice
C. Altered mental status
D. Fevers with chills

6. Hypertriglyceridemia is the third most common cause of acute pancreatitis


and usually the serum triglyceride levels are more than: (P2091)
A. 150mg/dl
B. 600mg/dl
C. 400mg/dl
D. 1000mg/dl

7. Portal hypertension is defined as the elevation of the hepatic cenous


pressure gradient and the most common cause is liver cirrhosis. The NORMAl Portal
venous pressure is:
A. <5 mmHg
B. 20mmHg
C. 5-10mmHg
D. None of the above

8. Turcot childs Pugh score system predicts the likelihood of major


complications of cirrhosis and its prognosis, it includes the following laboratory
determinanats except:
A. Albumin
B. SGPT/ALT
C. Protime
D. Bilirubin

9. The ultimate clinical manifestations of H.Plylori infection include the


following EXCEPT:
A. Lymphoma
B. Gastric cancer
B/C C. Duodenal ulcer
D. Gastric carices.

10. Gold standard for testing H. Pylori is:

A. Serology
B. Rapid urease test
C. Urea breath test
D. Histopathology

11. when the examiner asks the patient to close his eyes right eye does not
close with eyeball rolling up accompanied by flattening of the nasolabial fold upon
smiling. When asked to raise his eyebrows forehead is not wrinkled. Where is the
lesion ?

A. Trigeminal nerve
B. Facial Nerve
C. Upper motor neuron
D. Glossopharyngeal

12. Clinically, rest tremor, rigidity, bradykinesia(Slowing), and gait


impairment are the “cardinal features” of this disease :

A. Alzheimer’s
B. MultiSystem Atrophy
C. Parkinson’s
D. Cerebrovascular Disease

13. In parkinson’s disease, deficiency of this neurotransmitter leads to


increased firing of neurons in the subthalamic nucleus and globus pallidus interna,
excessive inhibition of the thalamus, reduced activation of cortical motor systems,
and the development of parkinsonian features:
A. Glutamate
B. Dopamine
C. Acetylcholine
D. GABA

14. This can manifest as young as the third decade, but it is the most
common cause of dementia in the elderly.

A. Creutzfeld Jacob Disease


B. Dementia with lewy Bodies
C. Alzheimer’s
D. Cortico basal ganglionic Degeneration

15 U.L. 20 year old Medical student complains of chronic head pain


characterized by bilateral, tight, band like discomfort which typically builds slowly,
fluctuates in severity, and may persist more or less continuously for many days. It is
not accompanied by nausea, vomiting, photophobia, phonophobia,osmophobia and
aggravation with movement :

A. Migraine
B. Tension Type headache
C. Cluster Headache
D. Hemicrania continua

16. Miss. G.H. 25 year old call center agent complains of repeated attacks of
headache chatacterized as throbbing, unilateral, severe , agitated by movement
accompanied by nausea and vomiting with episodes lasting for 1-2 days. PE and
Neuro Exam is unremarkable. What is most likely diagnosis ?

A. Stroke
B. Neuralgiform headache
C. Migraine headache
D. Cluster headache

17. This accounts for 50% of new cases of epilepsy in patients older than
age 65
A. Subdural hematoma
B. CNS Tumor
C. Degenerative diseases
D. Cerebrovascular disease

18. Rate of decrease in cerebral blood flow(mL/100 g tissue per minute)


that will cause infarction within an hour:
A. <16-18
B. <20
C. <100
D. Zero

19. The most common cause of cerebral embolism


A. Nonrheumatic atrial fibrillation
B.MI
C.Rheumatic heart disease
D.Ischemic cardiomyopathy

20. Neuromuscular disorder characterized by weakness and fatigability of


skeletal muscles, cause by a decrease in the number of available acetylcholine
receptor(AChRs) at neuromuscular junctions due to an antibody-mediated
autoimmune attack.
A.Guillain Barre syndrome
B.Multiple Sclerosis
C.Grave’s disease
D.Myasthenia gravis.

21. What is the main stay of treatment in most prolactinomas ?


A. dopamine agononist
B. dopamine antagonist
C. transcranial surgery
D. transsphenoidal surgery
E. external radiation therapy

22. this type of acquired hypopituitarism is an endocrine emergency that may result
in severe hypoglycemia, hypotension and shock and CNS hemorrhage and death.
A. empty sella
B. lymphocytic hypophysitis
C. pituitary apoplexy
D. pituitary dysplasia
E. pituitary adenomas

23. in grave s disease which is an auto immune disease that cause hyperthyroidism
the auto antibody mimics the action of which of the following hormones ..
A. TRH ( thyroid releasing hormone )
B. TSH ( thyroid stimulating hormone)
C. reverse t3
D. t3 (triiodotyronine )
E. t4 (thyroxine )

24. which of the following cause of thyrotoxicosis should not treated with radioactive
iodine ?
A. Thyrotoxicosis factitis
B. graves disease
C. toxic adenomas
D. toxic multinodular goiter
E. functioning thyroid cancer metastases

25. which of the following is false regarding treatment of hypothyroid patient ?


A. the usual replacement dose of levothyroxine in patient without residual thyroid is
1.6 mcg/kg ibw/day
B. levothyroxine should be tacken 30 min before breakfast on an empty stomach
C. ferrous sulfate, calcium supplements and proton pump inhibitor can be taken
along with levothyroxine tablet
D. because T4 has long half life (7 days )patiens who miss a doss can be advise to
take two doses of the skipped tablets at once
E. the dose is adjusted based on tsh level measured 2 month after initiating
levothyroxine

26. what is wolff-chaikoff effect ?


A. Thyroid hormone synthesis become excessive as a result of increased iodine
exposure
B. excess iodine exposure transientely inhibit iodide organification decrease thyroid
hormone synthesis
C. biosynthesis effect cause by incapacity of the thyroid to secret adequate amount
of hormone leading to increase to leading to increased tsh and large goiter
D. displacement of bound thyroid thyroid hormone thus increasing free hormone pool

27. manang bola is 54 yo female weight 140 ibs and 5’4 in height ,her bmi is
24kg/m2 according to clinical guidelines on the identification evaluation and
treatment of overweight and obesity in adult u.s department of health and human
service her BMI falls under the classification ?
A. underweight
B. normal weight
C. overweight
D. obesity
E. extreme obesity
28 R.R is a sedentary 45yo female with a history of polycystic ovarian syndrome her
physician orderd fasting plasma glucose for diabetes screening. She come back with
FPG=116mg/dl (6.4mmol/L) based on result she now has .
A. normal glycemic status
B. diabetes mellitus type 1
C. diabetes mellitus type 2
D. pre diabetes

29. which of the following insulin is short acting insulin ?


A. NPH
B. insulin detemir
C. insulin glargine
D. degludec
E. regular insulin

30. the sign pathognomic of hypocalcemia ?


A. Chadwick’s sign
B. dalrymple sign
C. trousseau’s sign
D. von graffe’ sign

For Question # 31-34 refer to the case below

Miss E.R. 20Y/O F present with 3 day history of sore throat and malaise. this is
associated with pain and swallowing but denies any cough.She recelved all her
childhood immunization P.E.reveals tender anterior cervical lymphadenopathy and
pharyngeal edema but no exudate.Temperature is 38.3˚C .The rest of the exam is
unremarkable.

31. What is the most likely diagnosis of the above case?


A. Community Acquired Pneumonia
B. Viral Pharyngitis
C. GAS Pharyngitis
D. Pharyngoconjunctival fever

32. Although no single clinical finding reliably distinguishes viral from bacterial
pharyngitis, Centorcriteria is used to reliably determine the likelihood of bacterial
pharyngitis. This criteria is composed of the following except:
A. fever D. tonsillar swelling/exudatesB. age
E. presence of cough
C. tender cervical lymphadenopathy

33.What remains to be the diagnostic glod standard for the diagnosis of the above
case?
A. throat culture C. ASO titer
B. bllod culture D. ELISA
34. What is the treatment of the case above?
A. Acyclovir 800mg 5x/d for 7 days
B. Penicillin V 500 mg bid, PO× 10 days
C. Amoxycillin 500 mg bid, PO ×7 days
D. Co-Amoxiclav 1.2 g q8h, IV

35. Based on the CPG for the Diagnosis, Treatment, Prevention and Control of
Tuberculosis in Adult Filipinos 2016 UPDATE proper term for a patient from whom a
biological specimen is positive by smear microscopy, culture or WHO-approved rapid
diagnostic test Xpert MTB/RIF:
A. Presumptive TB
B. Clinically diagnosed of TB
C. Bacteriologically confirmed case of TB
D. Case of pulmonary TB

36. According to the CPG for the Diagnosis, Treatment, Prevention and Control of
Tuberculosis in Adult Filipinos 2016 UPDATE case of PTB is already considered

Bacteriologically confirmed if;

A. At least one(1) sputum smer is positive for acid-fast bacilli


B. Two(2) out of three are positive for acid-fast bacilli
C. Three(3) out of the 3 specimen are positive for acid-fast bacilli
D. Two(2) out of the 2 specimen are positive for acid-fast bacilli

37. What is the effective treatment regimen for new PTB cases without risk factors
for drug resistance?
A .2HRZE/10HR C. 2HRZES/1HRZE/9HRE
B. 2HRZE/4 HR D. 2HRZES/1HRZE/5HRE

38. This remains as the gold standard in the diagnosis of drug resistant
Mycobacterium tuberculosis A. Xpert MTB/RIF
B Line Probe Assay(LPA)
C. Direct Smear Sputum Microscopy
D. Lowenstein Jensen method

39. Mr. U.L, 65y/o on PE has is alert with the following vital signs. BP 120/70mmHg
PR=110 bpm, RR=26 breaths/min, Temp=39˚C not in any form of pain. Base on the
data given patient has:
A. Septic Shock C. SIRS
B. Server Sepsis D. Refractory septic shock
40. in an immunocompetent adult having sever sepsis with known allergy to β-tactam
agents management includes :
A. Ciprofloxacin 400 mg q 12h, IV plus Vancomycin 15 mg/kg q 12
B. Cefepime 2g q 12h, IV
C. Piperacillin tazobactam 3.375g q4-6h, IV
D. Imipenem-Olastatin 0.5g q6h, IV

41. Mrs. M.L., >24 y/p, consulted at the E>R. because of cough and fever. She
claims the 38®-39 ®C fever started 8 days PTC. She self- medicated with
Paracetamol 500mg which afforded temporary relief. 5 days PTC, she started having
cough productive of yellowish , non- blood streaked sputum. This was followed by
chest pain on the left described as sharp and made worse by coughing. On P>E>
BMI= 22kg/m2 ;BP=100/80, HR= 100bpm ; RR=31 breaths/min, T= 38.9®C.
HEENT: unremarkable; no neck vein engorgement; C/L : symmetric chest
expansion, increased tactile fremitus left, dullness left base. What is your most
probable diagnosis at the time of examination ?
A. Pulmonary Embolism C. Acute Sinusitis
B. Pneumonia D. Left sided heart failure

42. Mr. S.D., 47 y/o, comes in because of cough of ½ month duration . it started with
fever moderate grade by palpitation late accompanied by cough with difficulty to
expectorate sputum. PMHx: (+) Diabetes controlled with Mewtformin 500 mg OD and
adherence to diet prescribed. ON P. E. BMO= 23.8 kg/m 2 ; BP= 100/80mmHg , HR=
120 bpm; RR= 28 breaths/min, T= 39.0® C. HEENT: unremarkable; no neck vein
engorgement; C/L : symmetric chest expansion, equal tactile fremitus , dullness left
lung field, mid to base , bronchial breath sounds left base, rales on left base.
Cardiovascular un remarkable. What is your most probable diagnosis at the time of
examination ?
A. Exacerbation of Asthma C. CAP, Low Risk
B. CAP, Moderate Risk D. CAP, High Risk

43. In a patient with an impression of community acquired pneumonia which of the


following would put him at a high risk CAP category?
A. BP 90/60 mmHg, RR= 36 breaths/ min, HR= 128bpm T=40®C
B. Uncontrolled Diabetes
C. Pleaural effusion
D. BP=80/50mmHg
44. Most common typical bacterial pathogen of community acquired pneumonia :
A. Streptococcus pneumonia
B. Mycoplasma pneumonia
C. Haemophilus influenza
D. All of the above

45. Sister M.J., 39 y/o present with a 10 days history of nasal congestion rhinorrhea.
This is accompanied by maxillary facial pain which is made worse on bending
forward, discomfort in the upper molar and yellowish nasal discharge. On P. E. BP=
100/80 mmHg, HR= 80bpm,; RR=16 breaths/min, T=38.6®. HEENT: tenderness on
pressure over maxillary is noted. No neck vein engorgement; C/L: symmetrical chest
expansion equal tactile fremitus, resonant, clear vesicular breath on all lung fields.
What Is your most probable diagnosis at the time of examination ?
A. Sinusitis C. Left- sided heart failure
B. Acute Bronchitis D. Pneumonia

46. Me. U. P., 28y/o, barangay health worker from Vigan, Ilocos Sur develop cough
and fever of 2½ week duration. Though he is a non- smoker , he claims because of
the bulk of patient in the health clinic he often eats 2ₓa day only. He also reports
easy fatigability , anorexia and weight loss . His cough is productive of yellowish
sputum, non blood streaked . ON P.E. BMI=16kg/m2, BP= 110/80mmHg, HR= 68
bpm, RR= 20 breaths/ min, T= 37.9®C. HEENT: unremarkable, no neck vein
engorgement; C/L: symmetric chest expansion, equal tactile fremitus, resonant, clear
breath sounds. What is your most probable diagnosis at time of examination ?
A. Left-sided heart failure C. Acute sinusitis
B.Pneumonia D. Pulmonary Tuberculosis

47. Y.H. 24, M presents with redness of both eyes accompanied by sore throat. PR
reveals T=38.7®C, generalized hyperemia OU, conjuctival follicles OU, (+)
pharyngeal follicle; non tender preauricular lymphadenopathy. What is the most
common etiology of the probable zuse of the disease?
A. Neisseria gonorrhea
B. Adenovirus
C. Staphylococcus
D. Chlamydial Trachomitis Serotype A-C

48. Most likely condition consistent with an analysis revealing a synovial fluid that is
turbid with decreased viscosisty, decreased protein & glucose content, WBC:
100,000µ/l , PMNs 90%, culture : G(+) cocci in clusters:
A. gonococcal arthritis C. Psoriatic arthritis
B. reactive arthritis D. non gonoccocal arthritis

49. Which is consistent with the epidemiology of asthma?


A. Asthma affects twice as many males than females during adulthood
B. Death is common in asthma
C. Atopy is the most important risk factor for the development of asthma
D. Asthma affects more females than males during childhood

50. Which of the following cells are important in initiating the acute
bronchoconstriction response to allergens?
A. Eosinophils
B. Mast cells
C. Neutrophiles
D. Macrophage and dendritic cell

51. What is the most common trigger for asthma ?


A. Exercise
B. Food allergy
C. Upper respiratory tract viral infection
D. Air pollution

52. What is expected in the spirometry of an asthmatic patient ?


A. Airflow limitation with good reversibility
B. Reduced FEV1
C. Reduced PEFR
D. Normal FEV1/FVC

53. Which of the following medications are used for moderately persistent asthma
?
A. Oral corticosteroids
B. Low dose ICS
C. High dose ICS
D. Montelukast

54. Which of the following parameter is most informative for restrictive ventialtory
defects ?
A. Forced vital capacity
B. FEV1
C. FEV1/FVC
D. FEF 25-75%
55. Which is not an established risk factor for the development of obstructive lung
disease ?
A. Previous TB infection
B. Cigarette smoking
C. Marijuana use
D. Motor vehicle exhaust

56. what is a characteristic feature of chronic bronchitis ?


A. Expiratory wheezing
B.Mucus Hypersecretion
C. Easy fatigability
D. Air trapping

57. Patient 15 year old and above, who has any of the following: cough at least 2
weeks duration: unexplained cough of any duration in a close contact of a
known active TB case or CXR finding suggestive of PTB with or without
symptos is :
A. Presumptibe TB
B. B. Bacteriologically confirmed case of TB
C. Clinically diagnosed case
D. Case of pulmonary TB

58. Which is not a sign of pulmonary hyperinflation on chest xray ?


A. Widened mediastinum
B. Increased retrocardiac space.
C. Flattened diapharagm
D. Hyperlucency

59. What is the gold criteria for diagnosing COPD ?


A.Scooping of the flow volume loop
B.Normal flow volume loop
C. Low postbronchodilator FEV1
D. Post bronchodilatior FEV1/FVC <0.70

60. The 2013 cholesterol guideline focused on 3-hydroxy-3methylglutaryl


coenzyme A reductase inhibitors rather than other classes of lipid modulating
drugs. The cholesterol guideline defined four statin benefit groups except:

A. all individuals who have clinical atherosclerotic cardiovascular


disease(ASCVD), therefore considered “secondary prevention”
B. Those with LDL cholesterol > or equal 190mg/dl without a secondary cause
C. Individuals with diabetes without established cardiovascular disease who
are 40-75 years old and havd LDL cholesterol of 70-189 mg/dl
D. Those without established ASCVD without diabetes who are 40-75 years
old and who have LDL cholesterol of 70-189 mg/dl
E. Those without established ASCVD without diabetes who are 40-75 years
old with calculated ASCVD risk > or equal5%

(61) Which of the following is the correct sequence of mecganism and manifestation
of ischemia :-
A.Endothelial dysfunction, diastolic dysfunction, decreased segmental perfusion ,
regional wall motion abnormality , systolic dysfunction
B. Endothelial dysfunction, systolic dysfunction, decreased segmental perfusion ,
regional wall motion abnormality , diastolic dysfunction
C. Endothelial dysfunction, systolic dysfunction , regional wall motion abnormality ,
diastolic dysfunction,decreased segmental perfusion
D. Endothelial dysfunction, decreased segmental perfusion, regional wall motion
abnormality , diastolic dysfunction,systolic dysfunction
E. Endothelial dysfunction, decreased segmental perfusion, regional wall motion
abnormality , systolic dysfunction,diasolic dysfuntion

(62) Most common cause of secondary hypertension is


A.Obesity and metabolic syndrome
B.renal parenchymal disease
C.renovascular hypertension
D.Primary aldosteronism
E.Cushing’s Syndrome

(63)which of the following is not common predisposing factors of deep vein


thrombosis?
A.obesity
B.Systemic arterial hypertension
C.chronic obstructive pulmonary disease
D.Blood transfusion
E.NOA

(64)The following statements are TRUE regarding aortic aneurysm except:-


A.saccular aneurysm affects the entire circumference of a segment of vesse,within a
fusiform aneurysm involves only a portion of the circumference
B.Most common pathologic condition associated with degenerative aortic aneurysm
in atherosclerosis
c.medial degeneration is the most common pathology associated with ascending
aortic aneurysms
D.atherosclerosis is the condition most frequently associated with aneurysms of the
descending thoracic aorta
E. Most thoracic aortic aneurysms are asymptomatic
(65) the infectious cause of aortic aneurysm include the following except :-
A.syphilis
B.M.tuberculosis
C. salmonella
D. Staphylococcal
E.E.coli

(66) DeBakey type 1 aneurysm is described as


A.an initial tear in the ascending aorta but invloves the desceding aorta as well
B.dissection is limied to ascending aorta
C.initial tear is located in the descending aorta with distal propagation of the
dissection
D. Dissection is limited to the arch and/or descending aorta ( Distal dissection )
E.NOA

(67) A disease of aorta which is called pulseless disease ?


A.giant cell arteritis
B. Takayasu’s arteritis
C.rhematic arterirtis
D. Idiopsthicaortitis

(68)the following statements describe atheroemblosim except:-


A.multiple small depostits of fibrin , platelet. And cholesterol debris embolize from
proximal atherosclerotic lesions
B.Digital vascular occlusion may result in ischemia and “ blue toe “ syndrome
C.localized areas of tenderness,pallor, and livedoreticularis occur at sited of emboli
D.Skin of muscle biopsy may demonstrate cholesterol crystals
E.NOA

(69) Which of the following not a prinicipal featured of tamponade(Beck’s triad) are :-
A.hypotension
B.soft or absent heart sounds
C,jugular venous distention with prominent Y descent but an absent X Descent
D. jugular venous distention with prominent X descent but an absent Y Descent

(70) At the emergency room,the intial test to be performed in triaging acute coronary
syndromes is :-
A.12-lead electrocardiogram
B.chest X-RAY
C.2D-Echocardiography
D.Cardiac enzymes
E.complete blood Count
71. 71. Most common clinical presentation of ischemic heart disease
A. Chronic stable Angina
B. Unstable Stable Angina
C. Acute myocardial infarction
D. Sudden cardiac death due to Arrhythmia
E. Congestive Heart failure

72. Pathologically, the most common anatomic substrate of chronic stable angina:
A. Head stenosing plaque of the coronary arteries
B.Coronary vasospasm
C.Thrombosis of coronary artery
D.Platelet aggregation
D.Normal Coronary Arteries

73. a patient with acute myocardial infraction with hypotention would belong to what
killip class:
A0 b 1 c 2 d3 e4

74 the patient with chest pain ecg shows st depression on lead avl,v5-v6,troponin
determination is podsitive what is your clinical diagnosis
A. chronic stable angina
B. unstable angina
C.NSTMI
D. STMI
E. ATYPICAL CHEST PAIN

75 THE MOST SPECIFIC CRITERIATION FOR SINUS RHYTHM ON ECG IS:


A. The P-wave should be identical preceding each QRS complex
B. The R-R Interval should be regular
C. The P-R interval should be less than 0.2sec
D. The QRS complex should be narrow
E. No ST segment deviation

76. The most common rhythm disorder in out of hospital sudden cardiac
death/arrest:
A. Asystole
B. Ventricular Tachycardia
C. Ventricular fibrillation
D. Pulseless electrical activity
E. Complete Heart block

77. The initial drug of choice for regular narrow QRS complex tachyarrythmia is:
A. Amiodarone
B. Lidocaine
C. Adenosine
D. Verapamil
E. any of the above

78. The vasopressor of the choice in patients with shock is:


A. Epinephrine
B. Norepinephrine.
C. Dopamine
D. Dobutamine
E. Nitroprusside

79. Which etiology of the shock is characterized by low systemic vascular resistance
due to vasodilation:
A. Hypovolemia
B. Cardiogenic
C. Sepsis
D. Obstructive shock
E. Pulmonary edema

80. in 2010, a collaborative effort between the American college of


rheumatology(ACR) and the European league against rheumatism(EULAR) revised
the 1987 ACR classification criteria for RA in an effort to improbe early diagnosis
with the goal of identifying patients who would benefit from early introduction of
DMARDs that take into account:
A. number and type of joint involvements
B. acute phase reactants and serology
C. duration of symptoms
D. radiographic joint erosions and subcutaneous nodules
E. All of the above
F. A,B,C

81) The following are the factors that increase the risk of developing rheumatoid
arthritis :
a) female sex E) A,B,C
b) (+) family history F) A,B,C,D,
c) smoking
d) allelic variation in HLA-DRBA1 gene

82) Joint manifestation of sever RA exhibiting hyperflexed PIP and hyperexteneded


DIP
A) swan-neck deformity C) Z deformity
B) Boutonnieres deformity D) Rheumatoid nodule

83) Synovial fluid analysis showing WBC>2000/ul but <50,000/ul with PMN >75%
and negative for growth on culture and abcense of crystals is compatibal with:
A) Osteo arthritis C) rheumatoid arthritis
B) infectious arthritis D) gouty arthritis

84) A palpable, less commonly audible, vibratory or cracking sensation elicited with
joint motion and with coarse joint crepitus indicating advanced cartilaginous and
degenerative changes
A) Dislocation C) crepitus
B) Subluxation D) Synovial effusion

85) Considered as a “red flag condition” or “ urgent condition” that must be


diagnosed promptly to avoid significant morbid or mortal sequelae
A) Septic arthritis E) A, B, C
B) Acute crystal induced arthritis F) A, B, C, D
C) Fracture
D) rheumatoid arthritis

86) Mrs QI.60 y/o, complains of right elbow pain, she reports pain on movement of
her right elbow. On examination as the examiner moves the involved joint there is no
report of pain, what would be part of your differential diagnoses???
A) Osteoporotic fracture D) Osteo arthritis
A/F B) Brusitis E) A, B, C, D
C) Polymyalgia rheumatic a F) A, B, C

87) Objective evidence of osteo arthritis on plain radiographs:


A) Osteophytes
B) Subchondral sclerosis
C) Subchondral cyst
D) Asymmetrical joint space narrowing
E) all of the above

88) D.F. 30y/o, male presents with a 6 days of fever and ankle pain, he consume
one bottle of beer a day at most. He prefers a fish over meat diet. PE reveals
BP=110/70, HR=98bpm, RR= 18 breaths/min, T= 39 ̊C, pain is 9/10.
Extrimities reveal : tender, erethematous, warm, flunctuant nodule on the
anterior left leg. Leg ankle: swollen, tender ,warm and erethematous with
decreased range of motion. Aspiration of synovial fluid from left leg reveals on
gram (+) cocci in cluster. What is the empiric drug of choice for this case ???
A) Penicillin G 2 million units IV every 4 hour for 2 weeks
B) cholchicine 0.6 mg tablet q 8h with subsequent tapering
C) Celecoxib 100 mg bid
D) Nafcillin 2 g q 4h, IV

89) Mr E.J. 40 y/o present with 8 h history of excruciating pain in his left big toe
(MTP1). He does not take any medication and does not have any other
medical condition. He drinks 5-6 bottles of beer every day. On P.E. reveals
T=38.1 C ̊ , Pulserate= 120/min , Respiratory rate= 20/min , BP=160/90 , BMI is
32. The joint is tender, swollen and erythematous. Pain is elicited in both
passive and active movement. What is most likely Diagnoses ?????
A) Septic arthritis C) Gouty arthritis
B) Tendonitis D) SLE

90) The following are anterior pituitary hormones except ?


A) ACTH B) ADH C) GH D) Prolactin

91. The following are posterior pituitary hormones except ?


A. ACTH B. ADH C. GH D. Prolactin

92. Which among the following statements is/are true ?


A. production of glucocorticoids and mineralocorticoids is under the control of the
HPA
B. production of glucocorticoids and adrenal androgens is under the control of the
HPA
C. production of glucocorticoids and adrenal androgens is under the control of the
HPA
D. production of glucocorticoids and adrenal androgens is under the control of the
RAAS
E. none of the above

93. Which among the following is the most important, if not, the only physiologic
action
of AVP/ADH ?

A. Reduce water excretion by promoting concentration of urine


B. Na retention
C. Potassium excretion
D. None of the above

94. Which of the following is the most useful physiologic marker of thyroid hormone
action ?
A. T4 B. T3 C. TBG D. TSH

95. What is the most abundant anterior pituitary hormone?


A.prolactin B. GH C. ACTH D.Oxytocin

96. Which of the following is the standard method for assessing long-term glycemic
control, reflecting the glycemic history over the previous 2-3 months?
A. FBS B. HBA1C
C. Both A and B D. None of the above

97. The National Institute for Health and Clinical Excellence In the United Kingdom
suggests the use of Metformin in the following situation, except?
A. should be used at a GFR of < 30ml/min
B. reduce dose if GFR is < 45ml/min
C. discontinued in hospitalized patients
D. on pxs. Receiving radiographic contrast material

98. Criteria for the diagnosis of DM except:


A. FPG of >/=126 mg/dl or >/=7 mmol/L
B. HBA1C >/= 6.5%
C. RBS >/=200 mg/dl or >/= 11.1 mmol/L plus symptoms of DM
D. 2-hr plasma glucose >/= 200 mg/dl or >/= 11.1 mmo/L
E. None of the above

99. Hypoglycemia is the most convincingly documented by Whipple’s triad which is


described as follows, except:
A. Symptoms consistent with hypoglycaemia
B. Low plasma glucose conc. Measured precisely
C. relief of symptoms after the plasma glucose is raised
D. None of the above

100. Tumor marker for hepatocellular cancer ?


A. CEA B. AFP C. HCG
D. PSA E. CD30
PARA and MICRO REVALIDA

1)Many of this type of bacteria do not possess superoxide dismutase and catalase ?

A. Obligate aerobes B.Obligate anaerobes


C.Facultative anaerobes D.Microaerophilic

2)It is the period when the bacterium is assessing the nutrients present in the
medium ,the bacterium does not divide but there is an intense metabolic activity .the
phase of bacterial growth is ?

A.Lag B.Log

C.Stationary D.Decline

3) A 10 –year old girl, with perianal pruritus, was brought by her mother to her
paediatrician. What is your most probable diagnosis?

A. Ascariasis B.Trichuriasis
C.Enterobiasis D.Hookworm infection

4) A 10 –year old girl, with perianal pruritus, was brought by her mother to her
paediatrician. What is your most probable diagnosis, what diagnostic procedure will
you request to support your diagnosis ?

A. Stool concentration technique b.Direct fecal smear


C.Harada –mori stool culture D.Cellulose tape swab

5) A 15 year old boy from davao del norte is noted to have pallor and malnutrition
.stool examination reveals an ovum with thin colorless cell wall .what is your
diagnosis ?

A. Ascariasis B.Trichuriassis
C.Enterobiasis D.Hookworm infection

6)Ova with flat bipolar plugs were seen in the stool specimen of a 16 year old
female with chronic diarrhea from taguda ilocos sur. What is your diagnosis ?

A.capilariasis B. hookworm infection


C. strongyloidasis D. ascariasis

7) the agg of te protozoa which causes rectal prolapse is

A .peanut shape egg B. with oncosphere and thick striated


shell

C. with deep cup like plts covering the surface of the shell
D. Chinese latent bipolar plug E. Chinese letters

8. A fisherman from davao oriental was rushed to near by hospital with chest pain,
persistant cough and hemoptysis as his main complaint.patient interview reveals
excessive drinking of native alcohol together with raw mountain crabs. Chest xray
demonstrated patchy, cloudy infiltration of lung with nodular shadows and calcified
spots. What parasitic infection can u consider?

a. pulmonary tuberculosis b. paragonimiasis

c. ascariasis d strongyloidiasis

9. a 24 year old abaca farmer from sorsogon with fever associated with signs and
symptoms of inflammation of the lymph glands was brought to the opd of the
philipine general hospital. Microscopic finding of the blood taken at 10 pm revealed
presense of larvae what is your diagnosis?

a. malariae b toxoplasmosis

c. filariasis d. angiostrogiloidiasis

10. TSST-1 toxin is example of

a. exfoliatin b enterotoxin

c. superantigen d. erythrogenic protien e. NOTA

11) 40y\o male overseas worker complaining of muscle pains, seek medical attention
upon his arrival in the philippines. Biochemical tests showed elevated creatinine
phosphokinase
Lactate dehydrogenase and myokinase levels. Results of cbc showed high blood
eosinophilia. What is your most probable diagnosis
A taeniasis B Capillariasis
C trichinosis D filariasis

12) The most commonly involved organ(s) in echinococcus granulosis is/are


A lungs B liver
C intestine D all of the above E A&B

13) individuals contact ascariasis lumbricoides by:

A-inhalation B-ingestion C-sexual contact D-insect bite


14) farmer complains of muscle pain in leg showed elevated creatinine,lactate
dehydrogenase,phosphokinase,myokinase levels. Result of CBC showed high blood
eosinophilia. Serologic test may confirm your diagnosis but since there is no
available serologic test what would you request to confirm your diagnosis?

A-skin test B-muscle biopsy

C-rectal biopsy D-skin scrapping

15. the most frequent entry site of virus is the

a. gi tract b. respiratory tract

c. intravascular d. cutaneous

e. NOTA

16. anaerobic infection is entertained in the presence of

a. sweet smelling discharge b. infection in proximity to the mucosal surface

c. growth in blood agar plate d. all of the above

e. A&B only

17. this tapeworm is equipped with four suckers and cone shaped armed rostallum
and its egg form membrane enclosed packets of 5 to 20 eggs

a. diphyllobothrium latum b. diphylidium caninum

c. hymenolepis nana d. taenia solium e. taenia saginata

18. An 8 year old girl complaint of diarrhea and abdominal pain, stool analysis
revealed barrel shaped eggs that appear to have plugs at each and parasite involved
is

a. ascaris lumbricoides b. ancylostoma duodenale

c. trichuris trichiura d. strongyloides stercoralis

e. enterobius vermicularis
19. the etiologic agent of pityriasis versicolor is

A.Malassezia b.trichosporon species

C. piedraia hortae D. none of the above

20. the following is DNA containing viruses

A. Adenoviruses B. Hepadanavirus

C. Coronavirus D. All of the above E. A&B

23) culture medium for fungi

A) chocolate agar B) sabouraud's agar

C) IsoVitalaX D) Inspissated egg medium e) NOTA

24) Anti microbial which exerts its action through inhibition of cell wall systhesis

A)aminoglycoside B)tetracycline

C)penicillin D)rifampicin E)NOTA

25) A 9 y/o girl presented with low grade fever, sore throat and malaise for 2 days.
She was noted to have dry cough and difficulty in breathing on PE patient was
febrile, tachycardic, tachypneic and with respiratory stridor, she had yellowish
leathery thick membrane extending to the uvula and soft palate. The above condition
is caused by

a. corynebacterium diptheriae b. haemophilus influenza

c. streptococcus pneumonia d. all of the above e.. A& B ONLY

26. Adult roung worm measuring 27 cm in length was seen in colon of 10 yr old boy
who died of pneumonia. Manifestation such as lung infiltration, asthematic attack
and edema of lips were documented before patient died. What is the most probable
parasitic infection can you identify in this patient?
A) trichuriasis B) Ascariasis
C) Capillariasis D) Hook worm infection

27) Fisher men from Davao oriental presenting with chest pain, persistent cough and
haemoptysis, he admitted to excessively drinking native alcohol and eating raw
mountain crabs what laboratory test you should work up?
A) Chest x-ray B) Sputum examination

C) Stool examination D) AOTA

28) 51 y/o farmer from surigao Mindanao was brought to the hospital with abdominal
pain ,frequent loos stool ,PE revealed hepatomegaly ,and tenderness in right upper
quadrant of the abdomen rectal biopsy shows egg with miracidia ,lab examination
shows eosinophilia, direct fecal smear shows no ova or parasite seen,based on lab
and clinical result what parasitic disease can you suggest?

A) Schistosomiasis B) leismanisis

C) paragonimiasis D) heterophydiasis

29. A wound culture showed yellow colonies which exhibit beta hemolysis, gram
positive cocci which are nonmotile on catalase test there was bubble the
microorganism most probably involved is

(a)Streptococcus pyogens b) staphylococcus aureus

c) staphylococcus epidermidis d) all e) a & b

30. systemic infection which is associated with tampon used by women , and
characterized by multiple organ failure ,hypotensive & desquamating skin which of
the following can present ?

a) scalded skin syndrome b) Toxic shock syndrome

c) sepsis d) toxic epidermal necrolysis e) NOTA

31. 21y/o sexually active female came to you with complaining 2 day history of
urinary frequency, urgency ,dysuria and evident of blood. She had no prior history of
UTI ,lab test reveals slightly elevated wbc 10,500/ul, urinalysis indicate innumerable
white cell. Culture shows >105 cfu lactose fermenting gram negative rod . most
likely asso etiologic agent is

A E.coli b. proteus mirabilis

c. klebisella pneumonia d. staphylococcus saprophyticus

32. An injured fire fighter developed wound infection, culture of site reveals gram
negative rod that is positive oxidase, produce bluish-green pigment, resistance to
antibiotic but susceptible to ticracillin and tobramycin. Most likely etiologic agent is

a. E.coli b. klebicilla pneumonia

c proteus mirabilis d pseudomonas aeruginosa


e. serratia marcescense

33) 30yr old female native of bulacan travelled to palawan for vacation after 2 week
goes to to doc complaining of fever chills what parrasitologic test u will do

A. stool test B. thick blood smear

C. thick and thin blood smear D. sputum test

34) this is a spiral shaped gram negative rod associated with antral gastritis, gastric
ulcer and gastric adenocarcinoma

a. campylobacter jejuni c. helicobacter pylori

b. aeromonas species d. NOTA

35) 50 year old male alcoholic was brought to your clinic expectoration of blood
tinged sputum. Microscopic study of Sputum reveals lancet shape gram +
organisms, which is most most likely :

a. streptococcus pneumonia b. klebsiella pneumonia

c. hemophilus influenza d mycobacterium tuberculosis

e.. pseudomonas aeruginosa

36) drug of choice for trichuris trichuria:

a. albendazole b. mebendazole c. metronidazole d. pyrantalpalmoate e.


NOTA

37) Resistance to tuberculosis is mediated by

a. cell mediated immunity b. humoral immunity

c. interferon d. NK cells e. none of the above

38) the leading cause of diarrhea in first 2 years of life among cases seen in hospital
and health centres worldwide is :

a. rotavirus b. e. coli

c. Entamoeba histolytica d. shigella flexneri e. adenovirus

39) innate immunity involves the following mechanisms/s:


A. phagocytosis B. Production of antibodies

C. T cells D. all of the above E. A &B ONLY

40) the antibody that has the ability to cross the placenta is :

A.igA B.igE C.igG D.igM E.igD

41) the hypersensitivity reaction that involves tgE- mediated release of histamine:

A. type 1 B. type 2

C. type 3 D. type 4

42) a 45 y/o farmer complained of worsening intense pain in his right leg 4 days after
sustaining an injury while plowing his field. The affected leg gradually become a
swollen and developed haemorrhagic blisters with crepitation’s. he underwent
extensive surgical debridement of the leg. But eventually died form septic shock the
infections was most likely caused by

a. clostridium tetani b. clostridium perfringens

c. clostridium botulinum d. clostridium difficile

e. none of the above

43. A 40 yr old woman has blurred vision and slurred speech, is Afebrile. Shes
known to be fond of eating home canned veggies and fruits. The microorganism
invoved is
a. clostridium tetani b. Clostridium perfringes
c. Clostridium botulinum d. Clostridium difficle e. NOTA

44. the following is/are Complication of Intestinal Amoebiasis except


a. Carcinoma b. perforations
c. Hemorrhage d.AOTA e. NOTA

45- sexually active 26 year old women consulted your clinic for vaginal itching and
purulent discharges. What is ur tentative diagnosis ??

A-gonorrhea B-trichomoniasis

C-candidiasis D-all of the above

46-trichomoniasis warrant which parasitologist exm..?

A-stool culture B-wet mount of vaginal fluid

C-pH test for blood sample D-ova and parasite fecal smear
47. The DOC of trichomoniasis

a. albendazole b. mebendazole

c. metronidazole d. praziquantel

48. the nematode whose larva does not undergo lung migration?

a. ascaris lumbricoides b. necator americanus

c. capilaria philipinensis d. strongyloides stercoralis

49) 30 year old women native of bulacan travelled to Palawan for vacation after 2
weeks she consulted her doctor with fever and chills as her main complain in
microscopic exam of thick and thin blood smear showed infected RBC with double
chromatin dot what is your most probable diagnosis?

A. not malariae b. plasmodium falciparum

c.plasmodium vivax d. plasmodium malariae

50) 6 month old baby was brought in for a fever of 1 day duration , she had found to
have temp 38.5 and hepatomegaly [3cm] and spleenomegaly [3cm] and in
microscopic examination revealed infected rbc with presence of band form stages
what is your probable diagnosis ?

a. not MALARIA b. plasmodium falciparum

c.plasmodium viva x d. plasmodium malariae

51. 30 year old pregnant women had a normal delivery but fetus was stillborn, CSF
examination shows +ve rods, the pathogen is the more likely:

a) Listeria monocytogenes
b) Gardenella vaginalis
c) Group B strep
d) Corneybacterium diphtheriae

52. Prevention of diptheria is brought about by:

a) Vaccination with toxoid


b) Passive immunization with antitoxin
c) Penicillin or erythromycin prophylaxis
d) AOTA
e) A & B only
53. A 49 y/o man from Manila recieved 4 units of PRBCs on January 15, while
undergoing hip replacement surgery. He was again hospitalized on February 1 with
fever, hypertension and renal failure. PBS confirmed malarial infection. The patient
has never traveled outside manila for 20 years. How did this patient acquire the
infection?

a) bite of mosquito
b) blood transfusion
c) relapse
d) infected syringe

54. The following virus/viruses is/are sexually transmitted:

a) HIV
b) hep B
c) papilloma virus
d) all of the above
e) A and B

55. Leptospirosis is acquired through:

a) Ingestion of contaminated food


b) Inhalation of microorganism
c) skin penetration of organisms
d) Sexual contact
e) NOTA

56. Staphylococcal Scalded skin syndrome a.k.a Ritter disease, is the separation of
the epidermis at the stratum granulosum this is caused by:

a) Protein A
b) Alpha toxin
c) Ab toxin
d) Erythrogenic toxin
e) Exfoliatin

57. 24 y/o male went through prosthetic valve implantation on his heart 5 year
ago.he has a great 3/6 systolic murmur on PE. You are entertaining acute infective
endocarditis. What is the most probable etiologic agent?
a) Staphylococcus aureus
b) Sthaphylococcus epidermidis
c) Streptococcus viridians
d) Streptococcus sanguls
e) C & D only

58. Psychotic indigent men with Hx of multi-substance abuse has been involuntarily
to hospitalized for 1 week because of persistent diarrhea. Stool sample was sent for
ova and parasites, revealing numerous granular, spherical thin walled cyst
measuring 10-20 um in diameter. Trichrome stain showed up to 4 nuclei in most of
the cyst, this findings are consistent with an infection by which of the following
organism:

a) Cryptosporidium parvum
b) Dientamoeba fragalis
c) Giardia lamblia
d) Entamoeba histolytica
e) Salmonella typhi

59. Wearing protective foot wear would not prevent transmission of which of the
following parasite?

a) Ancylostoma
b) Leptospira
c) Trichuris
d) Schistosoma

59. Taenia solium and trichinella spiralis can be prevented by eating properly cooked

a) Fish
b) Pork
c) Beef
d) Vegetables

61. Finding banana or sausage shaped gamate in peripheral smear is unique


characteristic of:

a) Plasmodium malaria
b) Plasmodium vivex
c) Plasmodium falciparum
d) Plasmodium ovale

62. Staphylococcal food poisoning is the result of

a) Heat labile enterotoxin


b) Ingestion of performed enterotoxin produced by staphylococcus aureus during
bacterial multiplication in contaminated food
c) Ingetion and multiplication of coagulase negative staphylococcus sp.
d) Multiplication of staphylococcus aureus in the small instestine.

63. A 19 year old student develops fever, carditis and migratory polyarthiritis. While
taking her history, the patient remembers that she had a “sore throat” 3 weeks
previously. A throat culture of patient yields normal flora only. The laboratory results
MOST CONSISTENT with this finding is that the patient has high titers of antibody
to:

a) Meningococcal lipooligosaccharide
b) Pneumococcal IgA protease
c) Pneumococcal polysaccharide
d) streptolysin O

64. 65 year old man with colonic tumor undergoes a bowel resection. 3 days later,
the patient develops fever and the drainage from the surgical site is foul smelling.
The skin near the site of the drainage is markedly erythematous, gram stain of the
drainage reveals gram positive cocci and gram negative rods. Aerobic culture of the
drainage yields streptococci. Gram negative rod seen on the gram stain was most
likely:

a) Bacteroides fragilis
b) Clostridium perfringens
c) E.coli
d) Pseudomonas aerugenosa

65) 50 year old rancher scraps his hand with clipper while shearing sheep, two days
later he notices ulcer surrounded by vessels at the site of wound, 8 days after the
injury, the wound develops a black necrotic center (eschar) surrounded by
edematous fluid. The rancher experiences mild systemic symptoms. The most likely
cause of the infection is:

a) Bacillus anthracis
b) Clostridium tetani
c) Neisseria meningitides
d) Streptococcus pyogens (group A streptococci)

66) 25 year old, previously healthy woman develops fever within 24 hour of delivery
of an infant born 6 weeks prematurely. Blood culture of the mother reveals gram
positive cocci that are catalase negative, beta haemolytic on blood agar, and
resistant to bacitracin the most likely cause of the bacteremia is:

a) Enterococci
b) Staphylococcus aureus
c) Streptococcus agalactiae (group B streptococci)
d) Streptococcus pneumoniae
67. HIV infected homosexual man presents with and Kaposi’s sarcoma. Which viral
genome is MOST LIKELY to be present in his tumor biopsy?

a) Herpes simplex virus type 1


b) Human Cytomegalovirus
c) Human herpes virus type 6
d) Human herpes virus type 8

68. Family of Measles Virus is,

a) Paramyxoviridae
b) Rhabdoviridae
c) Orthomyxoviridae
d) Flaviviridae

69. Family of hep A virus is:

a) Papovaviriade
b) hepadnaviridae
c) Parvoviridae
d) picornaviridae

70. Phenotypic charecteristic of steptococcus pyogenes is:

a) Alpha-haemolytic
b) Oxidase “+”
c) H2s-negative
d) Beta-hemolytic

71. Examination by dark field microscopy of scrapings from a genital ulcer of 19-year
old female reveals mobile, spiral-shaped organisms against a black background. The
MOST LIKELY diagnosis is:

a) Chancroid
b) Chlamydia trachomatis infection
c) Gonorrhoea
d) syphilis

72. Streptococcus pyogenes

a) Alpha
b) Catalase (+)
c) H2S negative
d) B hemolysis

73. Your patient, suffering from a bacterial infection, receives a penicillin injection
and almost immediately experiences respiratory distress and loses consciousness.
This rection is MOST LIKELY mediated by:
a) Complement
b) CD8+ cytolytic T cells
c) Macrophages
d) Mast cells

74. Which one of these statement explains the origin of the Rh antibodies in the
blood of an Rh negative mother?

a) They are naturally occurring in the blood of all Rh-negative persons.


b) They are received through mis-matched blood transfusions.
c) They are synthesized by mother in response to Rh-positive cells from her
child.
d) They are transferred from the Rh-positive father during intercourse.

75. A 21 yr old male with history of sexual contact 4days ago represents with painful
urination and purulent penile discharge. Which of the following is most likely:

a) primary syphilis
b) genital herpes
c) gonorrhea
d) lymphogranuloma venerum

76. a 1 and half year old baby developed fever coryza and cough followed 4 days
later by maculopapular rash starting from the hairline and becoming generalized.
Fever persisted.the most likely etiology is a/an:

a) RNA virus
b) DNA virus
c) Gram positive coccus
d) Gram negative bacillus

77. Amoebic abscess of liver is mostly found in the...

a) Left anteromedial lobe


b) Right posterosuperior lobe
c) Inferior lobe
d) Medial lobe

78. Hand- foot-and-mouth disease characterized by vesicular rash on the hands and
feet and ulcerations in the mouth,mainly in children is caused by:

a) Echovirus
b) Parainfluenza virus
c) Coxsackie virus Group A
d) Coxsackie virus Group B

79. the most common fungal opportunistic infection is caused by:


a) Aspergillus fumigatus
b) Cryptococcus neoformans
c) Candida albicans
d) Mucor

80. when symptoms to typhoid fever first become apparent, salmonella typi is most
frequently isolated from:

a) Feces
b) Urine
c) Blood
d) Bone marrow

81. The area in the body where salmonella typhi resides if one is a chronic carrier

a) small intestine
b) kidney
c) blood stream
d) gall bladder

82. A 3 years old boy was diagnosed to have meningitis,, CSF culture on chocolate
agar showed growth ofsmall gram negative rods. The most probable etiologic agent
is:

a) N.meningitis
b) S.aureus
c) Group B Streptococcus
d) H.influenzae type b

83. what parasite is associated with neoplasm of the biliary duct or cancer of the
liver?

a) Clonorchis sinensis
b) Fasciola hepatica
c) Opisthorchis felineus
d) Paragonimus westermani

84. what parasite produce infection that stimulate tuberculosis?

a) Clonorchis sinensis
b) Fasciola hepatica
c) Opisthorchis felineus
d) Paragonimus westermani

85. What is the infective stage of Schistosoma to man?

a) Cercariae
b) Embryonated egg
c) Metacercariae
d) Miracidium

86. Produces rectal prolapse in heavy infection especially among children.

a) Capillaria philippinensis
b) Trichinella spiralis
c) Trichuris trichiura
d) Ascaris lumbricoides

87. The complete infectious form of Hepatitis B is:

a) HBeAg
b) Dane particles
c) HBsAg
d) HBcAg

88. Which of the following virus/es contain/s hemagglutinin on its surface?

a) HEP B
b) HEP C
c) Influenza virus
d) All of the above

89.In their multiplication, they produce DNA which is integrated into cell genome.

a) Hapadnavirus
b) Reovirus
c) picornavirus
d) Herpesvirus

90. The Most common intrauterine viral infection, is caused by

a) CMV
b) Rubella
c) adenovirus
d) Herpes simplex

91. A 23 y/o office worker found out that his sexual partner had HIV infection. The
screening procedure of choice for HIV infection is:

a) PCR
b) HIV ELISA
c) western blot
d) southern blot

92. infection with this organism is the most common risk factor for cervical CA:

a) HIV
b) N.gonorrheae
c) HPV
d) HSV 2

93. most common cause of UTI in women is characterized by:

a) Urease production
b) Fruity odor on culture
c) Greenish metallic sheen
d) Swarming phenomenon

94. which of the following laboratory test can be used to monitor response to
treatment in syphilis?

a) FTA-ABS
b) VDRL
c) Darkfield microscopy
d) ELISA

95. An encapsulated fungi that causes meningitis

a) Histoplasma capsulatum
b) Cryptococcus neoformans
c) Candida albicans
d) Immitis

96. 20 y/o male developed fever, vomiting and loose appetite for 4 days followed by
jaundice, diagnosis of viral hepatitis is entertained.The results of the hepatitis profiles
are as follow: positive anti-HAV IgM, negative anti-HBsAg, negative anti-HBc IgM.
Which of following statements is correct?

a) patient has acute hepatitis A and is most likely a hepatitis B carrier


b) He has acute hepatitis B
c) He has acute hepatitis C
d) He does not have acute viral hepatits

97. Fetal hydrops is a prominent compilcation of maternal infection with:

a) Rubella
b) Parvo virus B19
c) Measles
d) Cytomegalovirus

98. The following virus/viruses exhibit/s latency in nerve ganglia:

a) smallpox virus
b) cytomegalovirus
c) chickenpox virus
d) adenovirus

99. Which of the following is a subcellular structure that is found only in gram
negative bacteria?

a) Endospore
b) Lipopolysaccharide
c) Mitochondria
d) Phosphatidylethanolamine

100. The following hepatitis B marker is indicative of active viral replication

a) HBsAg
b) HBeAg
c) Anti HBs
d) Anti HBe
LEGAL MEDICINE REVALIDA 2017

1. What is the charge for father who killed his newborn son?
a. Infanticide
b. Murder
c. Parricide
d. Homicide
2. Which event occurs in respiratory system when the person is nearly
dead?
a. Bluish discoloration
b. Something with 'Last 3 minutes'
c. Marked respiratory effort
d. Low blood pressure with low O2
3. Refers to injury, disease or combination of both that initiate disturbance
in physiological process that can be fatal.
a. Cause of death
b. Mechanism of death
c. Natural death
d. Manner of death
4. Physiologic disturbance of biochemical instability incompatible for life
a. Cause of death
b. Mechanism of death
c. Natural death
d. Manner of death
5. How much % of body weight should be lost to death?
a. 10
b. 15
c. 30
d. 40
6. a person cannot survive without food and water for 10 days but can not
survive with food yet without water for how long?
a. 15 days
b. 5-30 days
c. 30 – 50 days
d. 50-60 days
7. which of the following can prolong the length of starvation?
a. advanced age
b. body habitate
c. male sex
d. rapid metabolism
e. none of the above

8. mr.x died aue to natural death , his family reuired to obtain a death
certificate but no doctore are attain (him) dead body on dead bed. Who
else can issue his death certificate instead?
a. a neighbor who is a health officer in near by town
b. a barangay secretary
c. retire municipal health officer
d. department of health
9. a person who died of a communicablr disease , his/her body should be
buried with in ?
a. 3 days
b. 5 days
c. 24 hrs
d. 48 hrs
10. scar has inflammatory, redness and soft sensation….
a. few weeks to 2 months
b. 3weeks to 3 month
c. month to months
d. none
11. Name of the procedure for recording fingerprints
a. Dactylography
12. Differences between a male and a female pelvis except
a. In male, greater notch is narrow, in female; wide
b. Male obturator foramen is wider, female narrow
c. Male pubis is wider, female pubis is narrower
d. Male sacrum is short and narrow, female long and wide
13. Definition of plastic print
a. Indentations left without change
14. Determinants of sex
a. Presence of ovaries in female and testes in male
15. Determinants of sex
a. Presence of ovaries in female and testes in male
16.
17.
18.
19.
20.
21. If no physician is available, who among the following can sign the
death certificate?
a. City fiscal
b. DOH
c. Barangay secretary
d. ?
22. The most practical, useful and reliable means of processing
evidence
a. Photography
b. Sketching
c. Description
d. Mannequin method
23. Weapons, fingerprints and garments left at the crime scene are
examples of what type of evidence
a. Real
b. Ordinary
c. Corpus delicti(physical)
24. The following are device used to record the physiologic response in
individual except
a. Hypnotism
b. Word association
c. Lie detector
d. Psychological stress evaluation
25. Following is true about psycho-physiologic response
26.
27.
28.
29.
30.
31.
32.
33.
34.
35.
36.
37.
38.
39.
40.
41. most common market name of amphetamine except:
a. Golf ball
b. Pearly
c. Copilot
d. Peaches
42. Regulated drug:
a. Sedatives
b. Hypnotics
c. Amphetamine
d. Narcotics
43. Dangerous Drug Act:
a. Atropine
b. Codein
c. Hashish
d. Phenobarbitol
44. IndianHempt second name:
45. Regular taking of substance for physically well being
-(all of above answer—and drug addiction and dependence are same)
46. to announce death.
a. molecular death
b. cellular death
c. apparent death
47. type of injury produce laceration?
a. Heat
b. chemical
c. burn
d. tear forcing
e. change in atmosphere Pressure
48. type of wound in hollow organ??
a. deep wound
b. perforating
c. penetrating
d. Superficial
49. type of wound inside the body ??
a. deep
b. perforating
c. superficial
d. Penetrating
50. true crime of mutilation - intentionally cutting off any part of the body
51. A post-mortem wound differs from ante-mortem wound in that:
a. The wound edges gape
b. The tissues around the wound are deeply situated
c. There is a lot of blood present
d. There is no clotting of blood
52. The most important determinant of the wounding capacity of a bullet
is:
a. Bullet momentum
b. Bullet shape
c. Bullet caliber
d. Presence or absence of 'jacket'
53. Which of the following findings point to drowning as a cause of
death?
a. Blood chloride in the LV is the same as in RV
b. Washerwoman's hand and feet
c. Physical injuries indicative of a struggle
d. Marked congestion of the trachea and bronchi which are filled
with foreign bodies
54. Maria is married to and lives with Pedro. She gave birth after 181
days of their marriage. The child would be considered:
a. Natural
b. Illegitimate
c. Legitimate
d. Legitimated
55. Demonstration of impotence is important:
a. As a ground for divorce
b. As a ground for legal separation
c. As an evidence for alleged father in a paternal suit
d. As a basis for marriage disqualification
56.
57.
58.
59.
60.
61. not a crime of rape?
a. Virginity
b. women
c. intercourse shoude happen
d. forcefully happen
62. test of semen is human or not?
a. acid phosphate
b. brebio’s
c. Florence
d. precipiant
63. male physician intention is not good while doing brest examination
on female patient?
a. Malpractise
b. qualified seduction
c. act of lasciviousness
d. moral
64. last sentence to physician by dying patient about his killer?
a. hearsay decleration
b. dying decleration
c. inadmissible to court
d. ?
65. true about truth serum?
a. experimental
b. non volunteer after taking drug
c. inadmissble to court
d. ?
66.
67.
68.
69.
70.
71.
72.
73.
74.
75.
76. Person stabbed in abdomen > generalized peritonitis > dies of
peritonitis > stabbed wound considered
a. Murder
b. Proximal cause of death
c. Immediate cause of death
d. Underlying cause of death
77. Patient suffering from incurable disease > use physician to die >
physician give lethal dose > patient die > physician guilty for?
a. Murder
b. Homicide
c. ?
d. ?
78. Failure to initiate necessary management procedure to preserve life
in definition of euthanasia. This is called
79. Pregnant woman attempted suicide. Due to proper and timely
intervention, she didn't die but the fetus couldn't be saved. She is held
liable for
a. Attempted suicide
b. Abortion
c. Attempted suicide and abortion
d. No crime
80. Physical injury at the site of force and ? called
a. Coup
b. Contre coup
c. Coup contre coup
d. Locus ?
81. Brain is free and mobile, blot to the occipital side, contusion and
laceration occur at frontal lobe, what type of injury occur?
a. Direct injury
b. Contre coup
c. Coup contre coup
82. Length of the vertex of the skull to the symphasis pubis
a. ½ of the person’s height
b. 1/3 of the person’s height
c. 2/3 of the person’s height
d. 2/5 of the person’s height
83. Gunshot of entry except,
a. Smaller than messile
b. Border everted
c. Oval or round shape
d. Contusion collar is present
84. Gunshot of exit except,
a. Bigger than messile
b. Border everted
c. No shape/irregular
d. Contusion collar is absent
85. Contusion collar indicate/refer to:
a. Entry point
b. Distance from bullet was shot
c. Both
d. NOTA
86. electrical shock will lead to
a. ventricular fibrillation
b. respiratory failure
c. mechanical asphyxia
d. all
87. usual cause of medical malpractice
a. negligence
b. homicide
c. incompatible
d. all
88. proof of death given by:
a. cause
b. doubt
c. evidence
d. ?
89. presence of foreign body in abdomen after surgery, is a
a. doctrine of rese ipsa
b. doctrine of ?
c. rescue doctrine
d. doctrine of practice
90. brain death are consider when patient is
a. comatous
b. flat ECG
c. both
d. neither
91. true for dead on arrival
a. dead already when patient reach to hospital for ER
b. dead after 2 hr of hospitalization
c. dead after 2 days of hospitalization
d. dead on site
92. hospital not taking patient because not paying bills
a. can be leagally…
b.
c. patient abuse
d. all of the above
93. doctor can become expert witness by,
a. by using his sense
b. giving expert opinion
c. telling stories
d. all of the above
94.
95.
96. what is the true regarding the pharmacist sell drugs to the doctor ?
a. prohibited by low
b. not prohibited by low
c. not prohibited by low conduction with …….
d. AOTA
97. what is the prerequisite to practice as doctor according to Philippines
low ?
a. 21 year old ,medical certificate holder
b. Passed diploma of sub subspecialty
c. Qualified foreign medical examination
d. AOTA
98. Medical jurisprudents is branch of?
a. Medicine
b. Low
c. social
d. NOTA
99. Medical jurisprudents is deals with?
a. Legal medical case
b. (Long option)
c. legal ethics
d. NOTA
100. how long drowning the dead body floats?
a. 24 hrs.
b. 36 hrs.
c. 48 hrs.
d. 60 hrs.
ANATOMY REVALIDA 2017

I ) The antenor fontanelle also called


A Lambda B. Corona c. Brewna D.squamos E. None of the above

2) The heart located in the the


A Superwor mediastinum C Middle mednat
B. Antenor mediastumuun d Portenor medarstummn
E none of above

3 Which of the following statements is INCORRECT regarding the female breast


A Its amodified sweat gland B. Its lactiterous gland
C. Ligament of Cooper divides breast into 15-20 lobes
D Extends from 1 7 nbs
E. None of thr above

4 sternal angel to a e corresponds with the vertebra


A. TI b. T2 CT3 dT4 e none of the above

5. The accessory pancreatic duct maunly drains the


A Head of the pancreas None B Body of the pancreas
C Neck of the pancreas D. Tall of the pancreas pancreas
E none of the above

6. The most anterior Hilar structure found in the kxdney IS


A Renal artery b . Renal vern
C Ureter D Nerve e none of the above

7 The valve of Hester located in the


A common bile duct B. Pancreatic duct
C Cystic duct D. Ampulla of vater
E None of the above

8. The Camper's Lascia n the abdommal wall is contunuous with what structure un
the scrotum
A. Scarpa fascia B. Colle's fascia
C dartos fscia d transvalis fascia e none of d above

9. These ribs attach to the costal catilage above.


a.true ribs b. floating ribs ribs
c False nbs d all ribs e none of d above

10 The innermost layer of the comea is


A epithelium b. corneal stroma
c. endothelium D. Bowman's membrane e descent membrane
11 Congental duaphragmatic herma which located hemma postero
Laterally
A.Bochdalek herma E. None of above
b.morgagni hemia
c rickets hernoia
D. Spigelian henua

12. Clawhand deformity is second iary to damage of thus nerve,


A ulnar nerve B. Radial nerve c..median nerve d. axillary nerve e. none of d above

13. Which of the followung is not a capal bone


A Scaphold B. Lunate c. pisirom d. trapezoid e.none of d above

14 The most commonly dislocated joint


A hip joint b genohumeral joint c wrist joint d. elbow joint e. none of d above

15. The strongest layer of the antenor abdominal wall is


A A superficial fascia B. Intemal oblique muscle
C transversus fascia D. transversalis abdominis E. Note of the above

16. Whch of the follow ng not an anatomic construction of the esophagus


A upper esophageal sphuncter b Blow esophageal sphincter
C. Bronchochontric D Diaphramatic
E. None of the above

17 cells of the stomach mainly responsible for mucus secretion are mostly found in
A cardia b fundus c body d. pyloric anthrum e none of d above

18 Which of the following structer is not part of the colon


A. valvulae conniventes B appendes aepiplocia
c plica semicularis D haustra e None of the abwe

19 Which of the following is is not correct regarding normal anatomic position.


A Head and eyes directed forward
B Upper imbs by the sides with palms facang medially
C. Lower limbs together
d.Feet antenorty directed e none of the above

20. The movement by which the pad of the 1 digit brought to another digit t pad.
A. Reposition b. Pronation c. opposition d supination
E none of above

21 .which of the following is not a ventral cavity ?


A .pelvic cavity
B.vertebral cavity
C.Thoracic cavitv
D.Abdomnial Cavity
E.None of the above
22) cytoplasmic organelle mainly responsible for lipid and cholesterol synthesis.
A.Ribosomes
B.Endoplasmic Reticulum
C.Golgi apparatus
D.Proteasome
E.None of the above
( 23 )Cell membrane modification which aids in cell locomotion
A. Cilia
B. Flagella
C Microvilus
D.Microtubules
E None of the above

(24). Phase of mitosis charactenzed by the gradual colling of nuclear chromatin


giving rise to several individual rods
A.Prophase
B. Metaphase
C.Anaphase
D Telophase
E.None Of the Above
(25 ) Sensory innervatron of the tor 1/3 of the tongue is through,
A. chorda tympani nerve
B. glossohyarngeal nerve
C. hypoglossal nerve
D. lungual nerve
E. None of the above
(26) Adult venous system is derived from the followung embryonic structures.
EXCEPT
A. Vitelline vein
B .Umbilical veIn
C. Cardinal vein
D. SInus venosus
E.None Of Above
(27) Most common blood supply of the SA node comes from the
A. Left coronary artery
B. Right coronary artery
C.left circumflex artery
D Right marginal art
E.A and C
( 28) If the 3rd branchial arch fails to form or mature, which of the followIng cinical
manifestation would be observed?
A. No facial expression would be appreciated Hoarseness
B. Deafness
C. Hoarseness
D Decreased secreton of the parornd gland
E All of Above
(29 )The following layers of the anterior abdominal wall give rise to an equavalent
covering of scrotal and spermatic fascia, EXCEPT
A. Subcutancous fiscia
B. Extemal oblique
C. Intemal oblique
D. Trans versus abdominis
E. A and C
(30 )The following layers of the anterior abdominal wall give rise to an equavalent
covering of scrotal and spermatic fascia, EXCEPT
A. Subcutancous fiscia
B. Extemal oblique
C. Intemal oblique
D. Trans versus abdominis
E. A and C
(31) A stroke patient was found to have a necrotic focus on the brain specifically
identified as Brodrmam areas 44 and 45 .The patient is then expected to
A have difficulty in speaking but can comprehend
B. have difficulty understanding but can speak
C. not be able to hear and interpret sounds
D not be able to associate visual input to other mformation
E none of the above
(32 )The widest and most dilatable portion of the maie urethra
A Membranous
B. Spong
C.Penile
D Prostatic
E.B and c
(33 )The inferior parathyroid glands are denved from which branchial pouch
A.1st
B.2nd
C. 3rd
D.4th
E.C and D
(34 )This structure encloses the kidney and suprarenal gland and blends with the fascia
of IVC & aorta.
A Fibrous capsule
B. Penrenal fat
C Gerota's fascia
D Renal fascia
E.C and D
(35) A 46 year old patient came to emergency department due to gunshot wound.
During physical examinatuon, it was found out that the bullet entered the stenal area.
Which chamber of the heart will most probably be injured in this case?
A. Right atrium
B. Right ventricle
C Left ventricle
D Left atrium
Diaphragm
36 As the uterine artery passes from the internal iliac artery to the uterus It crosses
which of the following structures that sometime mistakenly ligated during pelvic surgery
?
A ovarian artery
B ovarian ligament
C uterine tube
D uterus
E round ligament of uterus

37 During a game, a 25 year old baseball player receives a severe blow to the
head that fractures the optic canal Which of the following parts of structures as most
likely to be damaged?
A Optic nerve and ophthalmic vein
B. Ophthalmic vein and ophthalmic nerve
C. Ophthalmic artery and optic nerve
D. Ophthalmic nerve and optic nerve
E ophthalmic artery and ophthalmic vein

38. What is the lining epithelium of the endocervical canal?


A Simple columnar
B Simple squamous
C Stratified squamous
D cubodal
E. Pseudostratified columnar

39 Communication of the and scala vestibule and scala tympani occur at the
A oval window
B round window
C helicotrema
D endolimphatic sac
E none of above
40 A 58 yo g8p8 underwent tahbho with bilateral lymphnode dissection for the high
grade cervical cancer few hour post surgery she complain that she can not feel her leg
on chest on your physical examination impaired right thigh adduction was observed the
iatrogenically damaged structure most likely the
A femoral nerve
B sciatic nerve
C obturator nerve
D pudendal nerve
E super gluteal nerve

41 The follow structures are part of the osteon exept


A osteocyte
B concentric lamellae
C canaliculi
D Volkmann canal
E a and b

42 The structure which has a head and a base that are united by two lumbs and is
connected to the oval window the
A incus
B Malleus
C States
D Stapedius
E Tensor tympani
43 All of the follow are contained in the vertebral canal. Except
A Spinal cord
C Dorsal nerve roots
D Ventral nerve roots
E Meninges
44 These granulocytes are described with bilobulatd nucleus' and plays roles in
fighting viral infection , as proved by the RNAses they contain
A basophils

B. eosinophil
C neutrophils
D NK cells
E Lymphocytes
45 Which among the following statements IS are true regarding difference between
Jejunum and ileum
A Jejunum is shorter
B ileum has more tat
C Jejunum is thicker
D B and C only
E All of the above
46 The rectum is divided into three (3) anatomical divisions. And its inferior part (distal
3rd)is being supplied a direct branch of?
A Inferior Mesenteric Artery
B sigmoidal artery
C Internal pudendal artery
D. A and C only
E. All of the above
47. Which among the following statements is true regarding the anatomy of the kidney?
A. 12 pyramids
B. 2 major calyces
C. 3 minor calyces
D. A and C only
E. All of the above
48. In the respiratory tract, what differentiates the terminal bronchioles from the
respiratory bronchioles?
A. the presence of alveoli in the respiratory bronchioles
B. Presence of greater number of smooth muscle in respiratory bronchioles
C. Presence of greater number of cilia in respiratory bronchioles
D. Lesser elastic fibers in respiratory bronchioles
E. All of the above

49. Which among the following statements is true regarding the round ligament of the
liver?
A. Aka ligamentum teres.
B From the Left umbilical vein
C A and C only
D b and c only
50. The following statements concerning the pancreas are correct except which?
A. The pancreas receive part of the arterial supply from the splenic artery.
B, The main pancreatic duct opens into the third part of the duodenum
C. The uncinated process of the pancreas projects from the head of the pancreas
D. The bile duct (CBD) lies posterior to the head of the pancreas

51. Melanocytes are derived from?


A. Tyrosine and leucine
B. Tyrosine and thyroxine
C. Cysteine and tyrosine

52. The following are presented by HLA-2?


A. Langerhans cell
B. Macrophages
c. Lymphocytes

53. Mammary ridges first seen at


A. 3- 4th week
B. 4- 5th week
C. 5- 6th week
D. 6- 7th week

54. 70% dry weight of the dermis is composed of


A. Keratin
B. Collagen
C. Lipo.....
D. None

55.which of the following sntns not correct regarding Normal ana position.
A. Head and eyes directed forward
B.upper limbs by the sides with palm medially
D.feet together
E.NOTA

57. lymph is similar to plasma, but it contain WBC and __________?


a. RBC b.platelet c. lymphocyte d. none

58. thoracic duct begin as dilated sac


a. cisterna chili b. vena cava c. aorta d. LN

59. the root f neck, thoracic duct , receiving blood EXCEPT :


A) right jugular B) subclavian
c) group of medial LN d) NOTA

60. the right lymphatic duct receive , EXCEPT :


a) drainage from left side b) right upper
c) right side of thoracic

d) NOTA , all are correct

61. Site of development of T lymphocyte..


A thymus B pineal gland
C thyroid gland D bone marrow

62. largest branch of celiac artery


A hepatic artery
B renal artery
C splenic artery
D gastric artery

63)Which ring covers respiratory and digestive track ?


A-Waldeyer ring

B-Palatine tonsil
C-Oropharynx
D-None of the above

64)Which prevent inward rotation and forward movement of tibia ?


A-Anterior cruciate ligament
B-PCL C-Collateral ligament

67) Carotid sheath contains following except:


A. Carotid Nerve C. Jugular vein
B. Vegus D. NOTA

68. Which muscle originate from spine of scapula and clavicular acromion and insert
into humerus?
A. Infraspinatus B. Teres minor
C. Deltoid D. Biceps

69 ) the embryologic remain of thyroglossal duct?

70) hypoglossus is innervated by??


a) hypoglossal nerve b) thyroglossal nerve
c) hypoglosuus nerve d) CN- 3

You might also like